Как решать дифференциальные уравнения: примеры решения диффуров (ДУ) в математике

Содержание

примеры решения диффуров (ДУ) в математике

Часто одно лишь упоминание дифференциальных уравнений вызывает у студентов неприятное чувство. Почему так происходит? Чаще всего потому, что при изучении основ материала возникает пробел в знаниях, из-за которого дальнейшее изучение диффуров становиться просто пыткой. Ничего не понятно, что делать, как решать, с чего начать?

Однако мы постараемся вам показать, что диффуры – это не так сложно, как кажется.

Основные понятия теории дифференциальных уравнений

Со школы нам известны простейшие уравнения, в которых нужно найти неизвестную x. По сути дифференциальные уравнения лишь чуточку отличаются от них – вместо переменной х в них нужно найти функцию y(х), которая обратит уравнение в тождество.

Дифференциальные уравнения имеют огромное прикладное значение. Это не абстрактная математика, которая не имеет отношения к окружающему нас миру. С помощью дифференциальных  уравнений описываются многие реальные природные процессы. Например, колебания струны, движение гармонического осциллятора, посредством дифференциальных уравнений в задачах механики находят скорость и ускорение тела. Также ДУ находят широкое применение в биологии, химии, экономике и многих других науках.

Дифференциальное уравнение (ДУ) – это уравнение, содержащее производные функции y(х), саму функцию, независимые переменные и иные параметры в различных комбинациях.

Существует множество видов дифференциальных уравнений: обыкновенные дифференциальные уравнения, линейные и нелинейные, однородные и неоднородные, дифференциальные уравнения первого и высших порядков, дифуры в частных производных и так далее.

Решением дифференциального уравнения является функция, которая обращает его в тождество. Существуют общие и частные решения ДУ.

Общим решением ДУ является общее множество решений, обращающих уравнение в тождество. Частным решением дифференциального уравнения называется решение, удовлетворяющее дополнительным условиям, заданным изначально.

Порядок дифференциального уравнения  определяется наивысшим порядком производных, входящих в него.

 

Решение уравнений

 

Обыкновенные дифференциальные уравнения

Обыкновенные дифференциальные уравнения – это уравнения, содержащие одну независимую переменную.

Рассмотрим простейшее обыкновенное дифференциальное уравнение первого порядка. Оно имеет вид:

Решить такое уравнение можно, просто проинтегрировав его правую часть.

Примеры таких уравнений:

Уравнения с разделяющимися переменными

В общем виде этот тип уравнений выглядит так:

Приведем пример:

Решая такое уравнение, нужно разделить переменные, приведя его к виду:

После этого останется проинтегрировать обе части и получить решение.

 

Математика

 

Линейные дифференциальные уравнения первого порядка

Такие уравнения имеют вид:

Здесь p(x) и q(x) – некоторые функции независимой переменной, а y=y(x) – искомая функция. Приведем пример такого уравнения:

Решая такое уравнение, чаще всего используют метод вариации произвольной постоянной либо представляют искомую функцию в виде произведения двух других функций y(x)=u(x)v(x).

Для решения таких уравнений необходима определенная подготовка и  взять их “с наскока” будет довольно сложно.

Пример решения ДУ с разделяющимися переменными

Вот мы и рассмотрели простейшие типы ДУ. Теперь разберем решение одного из них. Пусть это будет уравнение с разделяющимися переменными.

Сначала  перепишем производную в более привычном виде:

Затем разделим переменные, то есть в одной части уравнения соберем все «игреки», а в другой – «иксы»:

Теперь осталось проинтегрировать обе части:

Интегрируем и получаем общее решение данного уравнения:

Конечно, решение дифференциальных уравнений – своего рода искусство. Нужно уметь понимать, к какому типу относится уравнение, а также научиться видеть, какие преобразования нужно с ним совершить, чтобы привести к тому или иному виду, не говоря уже просто об умении дифференцировать и интегрировать. И чтобы преуспеть в решении ДУ, нужна практика (как и во всем). А если у Вас в данный момент нет времени разбираться с тем, как решаются дифференциальные уравнения или задача Коши встала как кость в горле или вы не знаете, как правильно оформить презентацию, обратитесь к нашим авторам. В сжатые сроки мы предоставим Вам готовое и подробное решение, разобраться в подробностях которого Вы сможете в любое удобное для Вас время. А пока предлагаем посмотреть видео на тему «Как решать дифференциальные уравнения»:

 

Дифференциальные уравнения с разделяющимися переменными

  • Дифференциальные уравнения, в которых переменные уже разделены
  • Дифференциальные уравнения, в которых требуется разделить переменные
  • Решить примеры самостоятельно, а затем посмотреть решения
  • Продолжаем решать примеры вместе

Дифференциальные уравнения, в которых выражение, зависящее от y, входит только в левую часть, а выражение, зависящее от x — только в правую часть, это дифференциальные уравнения с разделяющимися переменными, в которых переменные уже разделены.

В левой части уравнения может находиться производная от игрека и в этом случае решением дифференциального уравнения будет функция игрек, выраженная через значение интеграла от правой части уравнения. Пример такого уравнения — .

В левой части уравнения может быть и дифференциал функции от игрека и тогда для получения решения уравнения следует проинтегрировать обе части уравнения. Пример такого уравнения — .

Пример 1. Найти общее решение дифференциального уравнения

Решение. Пример очень простой. Непосредственно находим функцию по её производной, интегрируя:

Таким образом, получили функцию — решение данного уравнения.

Пример 2. Найти общее решение дифференциального уравнения

Решение. Интегрируем обе части уравнения:

.

Оба интеграла — табличные. Идём к решению:

Функция — решение уравнения — получена. Как видим, нужно только уверенно знать табличные интегралы и неплохо расправляться с

дробями и корнями.

Дифференциальные уравнения с разделяющимися переменными, в которых требуется разделить переменные, имеют вид

.

В таком уравнении и — функции только переменной x, а и — функции только переменной y.

Поделив члены уравнения на произведение , после сокращения получим

.

Как видим, левая часть уравнения зависит только от x, а правая только от y, то есть переменные разделены.

Левая часть полученного уравнения — дифференциал некоторой функции переменной x, а правая часть — дифференциал некоторой функции переменной y. Для получения решения исходного дифференциального уравнения следует интегрировать обе части уравнения. При этом при разделении переменных не обязательно переносить один его член в правую часть, можно почленно интегрировать без такого переноса.

Пример 3. Найти общее решение дифференциального уравнения

.

Это уравнение с разделяющимися переменными. Решение. Для разделения переменных поделим уравнение почленно на произведение и получим

.

Почленно интегрируем:

,

откуда, используя метод замены переменной (подстановки), получаем

или ,

поскольку левая часть равенства есть сумма арифметических значений корней. Таким образом, получили общий интеграл данного уравнения. Выразим из него y и найдём общее решение уравнения:

.

Нет времени вникать в решение? Можно заказать работу!

К началу страницы

Пройти тест по теме Дифференциальные уравнения

Есть задачи, в которых для разделения переменных уравнение нужно не делить почленно на произведение некоторых функций, а почленно умножать. Таков следующий пример.

Пример 4. Найти общее решение дифференциального уравнения

.

Решение. Бывает, что забвение элементарной (школьной) математики мешает даже близко подойти к началу решения, задача выглядит абсолютно тупиковой. В нашем примере для начала всего-то нужно вспомнить свойства степеней.

Так как , то перепишем данное уравнение в виде

.

Это уже уравнение с разделяющимися переменными. Умножив его почленно на произведение , получаем

.

Почленно интегрируем:

Первый интеграл находим интегрированием по частям, а второй — табличный. Следовательно,

.

Логарифимруя обе части равенства, получаем общее решение уравнения:

.

Пример 5. Найти общее решение диффференциального уравнения

.

Правильное решение и ответ.

Пример 6. Найти общее решение диффференциального уравнения

.

Правильное решение и ответ.

Пример 7. Найти общее решение дифференциального уравнения

.

Это уравнение с разделяющимися переменными

. Решение. Для разделения переменных поделим уравнение почленно на и получим

.

Чтобы найти y, требуется найти интеграл. Интегрируем по частям.

Пусть , .

Тогда , .

Находим общее решение уравнения:

Пример 8. Найти частное решение дифференциального уравнения

,

удовлетворяющее условию .

Это уравнение с разделяющимися переменными. Решение. Для разделения переменных поделим уравнение почленно на и получим


или
.

Записываем производную y в виде и получаем

Разделяем dy

и dx и получаем уравнение:

, которое почленно интегрируя:

,

находим общее решение уравнения:

.

Чтобы найти частное решение уравнения, подставляем в общее решение значения y и x из начального условия:

.

Таким образом частное решение данного дифференциального уравнения:

.

В некоторых случаях ответ (функцию) можно выразить явно. Для этого следует воспользоваться тем свойством логарифма, что сумма логарифмов равна логарифму произведения логарифмируемых выражений. Обычно это следует делать в тех случаях, когда слева искомая функция под логарифмом находится вместе с каким-нибудь слагаемым. Рассмотрим два таких примера.

Пример 9. Найти общее решение дифференциального уравнения

.

Это уравнение с разделяющимися переменными. Решение. Для разделения переменных запишем производную «игрека» в виде и получим

.

Разделяем «игреки» и «иксы»:

.

Почленно интегрируем и, так как в левой части «игрек» присутствует со слагаемым, в правой части константу интегрирования записываем также под знаком логарифма:

.

Теперь по свойству логарифма имеем

.

Находим общее решение уравнения:

Пример 10. Найти частное решение дифференциального уравнения

,

удовлетворяющее условию .

Это уравнение с разделяющимися переменными. Решение. Для разделения переменных поделим уравнение почленно на и получим


или
.

Разделяем dy и dx и получаем уравнение:


которое почленно интегрируя:

находим общее решение уравнения:

.

Чтобы найти частное решение уравнения, подставляем в общее решение значения y и x из начального условия:

.

Таким образом частное решение данного дифференциального уравнения:

.

Выводы. В дифференциальных уравнениях с разделяющимися переменными, как в тех, в которых переменные уже разделены, так и в тех, где переменные требуется разделить, существуют однозначные способы решения, на основе которых может быть построен простой алгоритм. Если недостаточно уверенно освоен материал по нахождению производной и решению интегралов, то требуется его повторить. Во многих задачах на путь к решению уравнения наводят знания и приёмы из элементарной (школьной) математики.

НазадЛистатьВперёд>>>

К началу страницы

Пройти тест по теме Дифференциальные уравнения

Всё по теме «Дифференциальные уравнения»

Порядок дифференциального уравнения и его решения, задача Коши

Дифференциальные уравнения с разделяющимися переменными

Однородные дифференциальные уравнения первого порядка

Линейные дифференциальные уравнения первого порядка

Дифференциальные уравнения Бернулли

Дифференциальные уравнения в полных дифференциалах

Дифференциальные уравнения второго порядка, допускающие понижение порядка

Линейные однородные дифференциальные уравнения второго порядка с постоянными коэффициентами

Линейные неоднородные дифференциальные уравнения второго порядка с постоянными коэффициентами

Поделиться с друзьями

Методические рекомендации для преподавателей математики и студентов средних специальных учебных заведений по теме «Дифференциальные уравнения»

I. Обыкновенные дифференциальные уравнения

1.1. Основные понятия и определения

Дифференциальным уравнением называется уравнение, связывающее между собой независимую переменную x, искомую функцию y и её производные или дифференциалы.

Символически дифференциальное уравнение записывается так:

F(x,y,y’)=0, F(x,y,y»)=0, F(x,y,y’,y»,.., y(n))=0

Дифференциальное уравнение называется обыкновенным, если искомая функция зависит от одного независимого переменного.

Решением дифференциального уравнения называется такая функция , которая обращает это уравнение в тождество.

Порядком дифференциального уравнения называется порядок старшей производной, входящей в это уравнение

Примеры.

1. Рассмотрим дифференциальное уравнение первого порядка

Решением этого уравнения является функция y = 5 ln x. Действительно, , подставляя y’ в уравнение, получим – тождество.

А это и значит, что функция y = 5 ln x– есть решение этого дифференциального уравнения.

2. Рассмотрим дифференциальное уравнение второго порядка y» — 5y’ +6y = 0. Функция – решение этого уравнения.

Действительно, .

Подставляя эти выражения в уравнение, получим: , – тождество.

А это и значит, что функция – есть решение этого дифференциального уравнения.

Интегрированием дифференциальных уравнений называется процесс нахождения решений дифференциальных уравнений.

Общим решением дифференциального уравнения называется функция вида ,в которую входит столько независимых произвольных постоянных, каков порядок уравнения.

Частным решением дифференциального уравнения называется решение, полученное из общего решения при различных числовых значениях произвольных постоянных. Значения произвольных постоянных находится при определённых начальных значениях аргумента и функции.

График частного решения дифференциального уравнения называется интегральной кривой.

Примеры

1.Найти частное решение дифференциального уравнения первого порядка

xdx + ydy = 0, если y = 4 при x = 3.

Решение. Интегрируя обе части уравнения, получим

Замечание. Произвольную постоянную С, полученную в результате интегрирования, можно представлять в любой форме, удобной для дальнейших преобразований. В данном случае, с учётом канонического уравнения окружности произвольную постоянную С удобно представить в виде .

— общее решение дифференциального уравнения.

Частное решение уравнения, удовлетворяющее начальным условиям y = 4 при x = 3 находится из общего подстановкой начальных условий в общее решение: 32 + 42= C2; C=5.

Подставляя С=5 в общее решение, получим x2 +y2 = 52.

Это есть частное решение дифференциального уравнения, полученное из общего решения при заданных начальных условиях.

2. Найти общее решение дифференциального уравнения

Решением этого уравнения является всякая функция вида , где С – произвольная постоянная. Действительно, подставляя в уравнения , получим: , .

Следовательно, данное дифференциальное уравнение имеет бесконечное множество решений, так как при различных значениях постоянной С равенство определяет различные решения уравнения .

Например, непосредственной подстановкой можно убедиться, что функции являются решениями уравнения .

Задача, в которой требуется найти частное решение уравнения y’ = f(x,y)  удовлетворяющее начальному условию y(x0) = y0, называется задачей Коши.

Решение уравнения y’ = f(x,y), удовлетворяющее начальному условию, y(x0) = y0, называется решением задачи Коши.

Решение задачи Коши имеет простой геометрический смысл. Действительно, согласно данным определениям, решить задачу Коши y’ = f(x,y)  при условии y(x0) = y0,, означает найти интегральную кривую уравнения y’ = f(x,y)  которая проходит через заданную точку M0(x0,y0).

II. Дифференциальные уравнения первого порядка

2.1. Основные понятия

Дифференциальным уравнением первого порядка называется уравнение вида F(x,y,y’) = 0.

В дифференциальное уравнение первого порядка входит первая производная и не входят производные более высокого порядка.

Уравнение y’ = f(x,y) называется уравнением первого порядка, разрешённым относительно производной.

Общим решением дифференциального уравнения первого порядка называется функция вида , которая содержит одну произвольную постоянную.

Пример. Рассмотрим дифференциальное уравнение первого порядка .

Решением этого уравнения является функция .

Действительно, заменив в данном уравнении, его значением, получим

то есть 3x=3x

Следовательно, функция является общим решением уравнения при любом постоянном С.

Найти частное решение данного уравнения, удовлетворяющее начальному условию y(1)=1 Подставляя начальные условия x = 1, y =1  в общее решение уравнения , получим откуда C = 0.

Таким образом, частное решение получим из общего подставив в это уравнение, полученное значение C = 0 – частное решение.

2.2. Дифференциальные уравнения с разделяющимися переменными

Дифференциальным уравнением с разделяющимися переменными называется уравнение вида: y’=f(x)g(y) или через дифференциалы , где f(x)  и g(y)– заданные функции.

Для тех y, для которых , уравнение y’=f(x)g(y) равносильно уравнению, в котором переменная y присутствует лишь в левой части, а переменная x- лишь в правой части. Говорят, «в уравнении y’=f(x)g(y разделим переменные».

Уравнение вида называется уравнением с разделёнными переменными.

Проинтегрировав обе части уравнения по x, получим G(y) = F(x) + C– общее решение уравнения, где G(y) и F(x) – некоторые первообразные соответственно функций и f(x), C произвольная постоянная.

Алгоритм решения дифференциального уравнения первого порядка с разделяющимися переменными

  1. Производную функции переписать через её дифференциалы
  2. Разделить переменные.
  3. Проинтегрировать обе части равенства, найти общее решение.
  4. Если заданы начальные условия, найти частное решение.

Пример 1

Решить уравнение y’ = xy

Решение. Производную функции y’ заменим на

разделим переменные

проинтегрируем обе части равенства:

Ответ:

Пример 2

Найти частное решение уравнения

2yy’ = 1- 3x2, если y0 = 3 при x0 = 1

Это—уравнение с разделенными переменными. Представим его в дифференциалах. Для этого перепишем данное уравнение в виде Отсюда

Интегрируя обе части последнего равенства, найдем

Подставив начальные значения x0 = 1, y0 = 3 найдем С 9=1-1+C, т.е. С = 9.

Следовательно, искомый частный интеграл будет или

Пример 3

Составить уравнение кривой, проходящей через точку M(2;-3) и имеющей касательную с угловым коэффициентом

Решение. Согласно условию

Это уравнение с разделяющимися переменными. Разделив переменные, получим:

 Проинтегрировав обе части уравнения, получим:

Используя начальные условия, x = 2  и y = — 3 найдем C:

Следовательно, искомое уравнение имеет вид

2.3. Линейные дифференциальные уравнения первого порядка

Линейным дифференциальным уравнением первого порядка называется уравнение вида y’ = f(x)y + g(x)

где f(x) и g(x) — некоторые заданные функции.

Если g(x)=0 то линейное дифференциальное уравнение называется однородным и имеет вид:  y’ = f(x)y

Если то уравнение y’ = f(x)y + g(x) называется неоднородным.

Общее решение линейного однородного дифференциального уравнения y’ = f(x)y задается формулой: где С – произвольная постоянная.

В частности, если С =0, то решением является  y = 0 Если линейное однородное уравнение имеет вид y’ = ky где k — некоторая постоянная, то его общее решение имеет вид: .

Общее решение линейного неоднородного дифференциального уравнения y’ = f(x)y + g(x) задается формулой ,

т.е. равно сумме общего решения соответствующего линейного однородного уравнения и частного решения данного уравнения.

Для линейного неоднородного уравнения вида y’ = kx + b,

где k и b— некоторые числа и частным решением будет являться постоянная функция . Поэтому общее решение имеет вид .

Пример. Решить уравнение y’ + 2y +3 = 0

Решение. Представим уравнение в виде y’ = -2y — 3 где k = -2, b= -3 Общее решение задается формулой .

Следовательно, где С – произвольная постоянная.

Ответ:

2.4. Решение линейных дифференциальных уравнений первого порядка методом Бернулли

Нахождение общего решения линейного дифференциального уравнения первого порядка y’ = f(x)y + g(x) сводится к решению двух дифференциальных уравнений с разделенными переменными с помощью подстановки y=uv, где u и v — неизвестные функции от x. Этот метод решения называется методом Бернулли.

 Алгоритм решения линейного дифференциального уравнения первого порядка

y’ = f(x)y + g(x)

1. Ввести подстановку y=uv.

2. Продифференцировать это равенство y’ = u’v + uv’

3. Подставить y и y’ в данное уравнение:   u’v + uv’ = f(x)uv + g(x) или u’v + uv’ +  f(x)uv = g(x).

4. Сгруппировать члены уравнения так, чтобы u вынести за скобки:

5. Из скобки, приравняв ее к нулю, найти функцию

Это уравнение с разделяющимися переменными:

Разделим переменные и получим:

Откуда . .

6. Подставить полученное значение v в уравнение (из п.4):

и найти функцию Это уравнение с разделяющимися переменными:

7. Записать общее решение в виде: , т.е. .

Пример 1

Найти частное решение уравнения y’ = -2y +3 = 0  если y =1  при x = 0

Решение. Решим его с помощью подстановки y=uv, .y’ = u’v + uv’

Подставляя y и y’ в данное уравнение, получим

Сгруппировав второе и третье слагаемое левой части уравнения, вынесем общий множитель u за скобки

Выражение в скобках приравниваем к нулю и, решив полученное уравнение, найдем функцию v = v(x)

Получили уравнение с разделенными переменными. Проинтегрируем обе части этого уравнения: Найдем функцию v:

Подставим полученное значение v в уравнение Получим:

Это уравнение с разделенными переменными. Проинтегрируем обе части уравнения: Найдем функцию u = u(x,c) Найдем общее решение: Найдем частное решение уравнения, удовлетворяющее начальным условиям y = 1 при x = 0:

Ответ:

III. Дифференциальные уравнения высших порядков

3. 1. Основные понятия и определения

Дифференциальным уравнением второго порядка называется уравнение, содержащее производные не выше второго порядка. В общем случае дифференциальное уравнение второго порядка записывается в виде: F(x,y,y’,y») = 0

Общим решением дифференциального уравнения второго порядка называется функция вида , в которую входят две произвольные постоянные C1 и C2.

Частным решением дифференциального уравнения второго порядка называется решение, полученное из общего при некоторых значениях произвольных постоянных C1 и C2.

3.2. Линейные однородные дифференциальные уравнения второго порядка с постоянными коэффициентами.

Линейным однородным дифференциальным уравнением второго порядка с постоянными коэффициентами называется уравнение вида y» + py’ +qy = 0, где pи q— постоянные величины.

Алгоритм решения однородных дифференциальных уравнений второго порядка с постоянными коэффициентами

1. Записать дифференциальное уравнение в виде: y» + py’ +qy = 0.

2. Составить его характеристическое уравнение, обозначив через r2, y’  через r, yчерез 1:r2 + pr +q = 0

3.Вычислить дискриминант  D = p2 -4q и найти корни характеристического уравнения; при этом если:

а) D > 0; следовательно, характеристическое уравнение имеет два различных действительных корня . Общее решение дифференциального уравнения выражается в виде , где C1 и C2 — произвольные постоянные.

б) D = 0; следовательно, характеристическое уравнение имеет равные действительные корни . Общее решение дифференциального уравнения выражается в виде

в) D < 0; следовательно, характеристическое уравнение имеет комплексные корни, Общее решение дифференциального уравнения выражается, в виде 

Примеры.

1. Найти частное решение дифференциального уравнения

Решение. Составим характеристическое уравнение

D>0,

Общее решение

Дифференцируя общее решение, получим

Составим систему из двух уравнений

Подставим вместо ,и заданные начальные условия:

Таким образом, искомым частным решением является функция

.

2. Найти частное решение уравнения

Решение

<0,

Общее решение

— частное решение.

IV. Практическая работа

Вариант 1

1. Составить уравнение кривой, проходящей через точку M(1;2) и имеющей угловой коэффициент .

2. Найти частные решения дифференциальных уравнений:

а)

б)

в)

г)

Вариант 2

1. Составить уравнение кривой, проходящей через точку M(2;1) и имеющей угловой коэффициент

2. Найти частные решения дифференциальных уравнений:

а)

б)

в)

г)

V. Ответы

Вариант 1

Вариант 2

1.

 1.

2. а)

2. а)

б)

б)

в)

в)

г)

г)

Дифференциальные уравнения

Одной из дисциплин, входящих в курс Высшей математики, является курс дифференциальных уравнений, решение которых у студентов традиционно вызывают трудности. В данной статье постараюсь показать примеры решения некоторых видов таких уравнений.

Итак, дифференциальным уравнением (иногда, студенты называют их любя – “дифуры”) называют уравнение, которое содержит неизвестные функции, их аргументы и производные от неизвестных функций по этим аргументам (или дифференциалы неизвестных функций). 

Подавляющее большинство задач в прикладных науках, если формулируют их на языке математики, приводят именно к различным дифференциальным уравнениям. Мы рассматриваем лишь обычные дифференциальные уравнения, одной из характерных особенностей которых есть то, что неизвестные функции в этих уравнениях зависят лишь от одной переменной.

Общий вид обычного дифференциального уравнения n — го порядка такой: F(x, y, y’,…, y(n-1), y(n)) = 0, где x — независимая переменная, y — неизвестная функция переменной x, а y, y’,…,y(n) — производные неизвестной функции по переменной x.

Порядком дифференциального уравнения называют порядок старшей производной, которая входит в это уравнение.

Решением дифференциального уравнения называют функцию y = φ(x), которая при подстановке в уравнение на место неизвестной функции превращает это уравнение в тождество. Решение дифференциального уравнения, заданное неявным соотношением, Ф(x,y) = 0 называют интегралом этого уравнения.

В этой статье будем употреблять термин проинтегрировать дифференциальное уравнение, которое означает найти все его решения. 

§1. Дифференциальное уравнение I-го порядка 

Общий вид дифференциального уравнения I-го порядка выглядит следующим образом:

F(x, y, y’) = 0 (1.1)

Если соотношение (1.1) решить относительно производной, как вариант дифференциала, то получим уравнение такого вида:

y’ = f(x, y) (1.2)

Такое уравнение называют дифференциальным уравнением, решенным относительно производной. Дифференциальное уравнение I-го порядка имеет, вообще говоря, не одно, а бесконечное множество число решений. Чтобы из этого множества решений выделить определенное решение, задают значение неизвестной функции y = y0  при некотором значении аргумента x = x0.

Условие, что при x = x0 функция упринимает заранее заданное значение y0, называют начальным условием. Мы это условие запишем в виде 

y|x=x0 = y0или y(x0) = y(1.3)

Проблему нахождения решения дифференциального y’ = f(x,y) уравнения, которое удовлетворяет начальному условию y(x0) = y0, называют задачей Коши.

Теорема 1.1. Если в уравнении y’ = f(x,y)  функция f(x,y)  и ее частная производная f’y(x,y)  непрерывны в некоторой области G плоскости Oxy, которая содержит точку (x0,y0), то существует и при этом единственное решение y=φ(x) такого уравнения, которое удовлетворяет условию y(x0) = y0.

Введем теперь еще несколько основных определений.

Определение 1.1. Общим решением (в дальнейшем, для краткости ОР) дифференциального уравнения I-го порядка называется функция

y = φ(x, C) (1.4)

которая зависит от одной произвольной постоянной С и удовлетворяет таким условиям:

1) она удовлетворяет уравнению при любом конкретном значении постоянной С;

2) каким бы не было начальное условие y(x0) = y0, всегда можно найти такое значение С = С0, так что функция y= φ(x, C0) будет удовлетворять этому начальному условию.

Замечание. При построении общего решения «дифура» очень часто приходят к соотношению вида

Ф(x, y, c) = 0 (1.5)

не решаемому относительно y.

Равенство Ф(x, y, c) = 0, которое неявно задает общее решение (в дальнейшем, для краткости ОР), называют общим интегралом (в дальнейшем, для краткости ОИ) дифференциального уравнения.

Определение 1.2. Частным решением дифференциального уравнения I-го порядка называется функцияy= φ(x, C0), которую получаем из его общего решения y= φ(x, C) при определенном значении C = C0.

Соотношение Ф(x, y, C0) = 0называют частным интегралом дифференциального уравнения I-го порядка. 

§2. Дифференциальные уравнения I-го порядка с разделяющимися переменными

Определение 2.1. Дифференциальное уравнение I-го порядка вида

φ(y)dy = f(x)dx (2.1)

называется уравнением с переменными, которые можно разделить.

Непосредственно (дифференцированием) устанавливается, что ОИ уравнения (2.1) является соотношение

∫ φ(y)dy = ∫ f(x)dx (2.2)

где — C=const.

Пример 2.1. Решить “дифур” 2y2dy = 3xdx.

Решение. Найдем неопределенные интегралы от правой и, конечно же, левой части

Легко увидеть, что это решение, при желании, можно записать в явной форме , но обычно его оставляют в той форме, в которой получили, кое-что упростив получим 4y3 = 9x2 + C.

Пример 2.2. Решить “дифур”  

Решение. Найдем неопределенные интегралы от правой и, конечно же, левой части

Поскольку C=const, то зачастую в такой форме решения для удобства записи, вместо C пишут ln |C|, а дальше выражение потенцируют

ln|y — 1| = ln|x| + ln C

ln|y — 1| = ln|Cx|

y – 1 = Cx

y = Cx + 1. 

Определение 2.2. Дифференциальное уравнение I-го порядка называется уравнением с переменными, которые можно разделить, если его правая часть является произведением двух функций, одна из которых зависит лишь от аргумента х, а вторая от неизвестной функции у:

 

Здесь мы считаем, что функция φ(x) определена и непрерывна для всех ϵ (a,b) а функция ѱ(y) определена и непрерывна и не равна нулю для всех ϵ (c,d).

Если переписать уравнение (2.2) в виде  , то левая часть зависит только от переменной у, а правая часть зависит только от переменной х, то есть переменные отделены. Тогда общий интеграл запишется в виде

,

где С=const.

Пример 2.3. Решить “дифур”

Решение. Перед нами уравнение с переменными, которые можно разделить,. Запишем производную в виде соотношения дифференциалов: y’ = dy/dx, умножим обе части уравнения на dx  и разделим на lny. В результате проделанной замены и “перемещения” переменных получим уравнение, в котором разделены переменные

После вычисления интегралов, имеем

y= eCx  ОР искомого уравнения.

Пример 2.4. Эффективность рекламы.

Пусть фирма продает продукцию B, про которую на момент времени tиз числа возможных клиентов знает лишь xклиентов. Далее, для увеличения продажи продукции, были сделаны рекламные объявления на радио и телевидении. Далее информация о товаре распространяется между клиентами через общение. После рекламы скорость изменения числа клиентов, которые знают о продукции B, пропорциональная не только числу клиентов, которые знают о товаре, но и числу клиентов, которые еще не знают.

Если допустить, что счет времени начинается после рекламных объявлений, когда о продукции узнало N/ɣ  человек, то получаем дифференциальное уравнением с переменными, которые можно разделить

При таких начальных условиях: x = N/ɣ , если t = 0. Здесь k— положительной коэффициент пропорциональности.

Интегрируя уравнение, имеем:

В экономической литературе это выражение называют уравнением логистической кривой.

С учетом начальных условий, получим

Замечание. Уравнение с переменными, которые можно разделить, можно также задать в симметричной относительно и y дифференциальной форме

M(x) · N(y)dx+ P(x) · Q(y)dy=0 (2. 4)

где функции M(x), P(x), N(y), Q(y) непрерывны соответственно в интервалах x ϵ (a,b), y ϵ (c,d).

Для нахождения решений необходимо разделить правую, (желательно, конечно) и левую части на произведение: N(y) · P(x).

и интегрируют полученное так соотношение

Если для x ϵ (a,b), y ϵ (c,d) функции P(x) и N(y) отличающиеся от нуля, то соотношение (2.6) является ОИ уравнения (2.4).

Пример 2.5. Решить “дифур” x(1 + y2)dx– y(1 + x2)dy = 0

Решение. Поступим также, как и в серии предыдущих примеров (разделим обе части уравнения на (1 + y2) · (1 + x2)

Интегрируя каждое из слагаемых (для этого не обязательно один из них переносить в правую часть), приравниваем сумму первообразных постоянной, которую обозначаем через ½ ln C, имеем:

Пример 2. 6. Решить “дифур” y’ + 2x2y’ + 2xy– 2x = 0.

Решения. Представим производные в виде соотношения dy/dxи далее все члены уравнения домножим на dx:

Сгруппируем члены с разными дифференциалами и вынесем за скобки дифференциалы.

(1 + 2x2)dx +2x(y– 1)dx = 0

В результате деления на (1 + 2x2) (y– 1). Получим:

Интегрируем каждое из слагаемых:

Сумму первообразных приравниваем постоянной:

тогда

– ОИ уравнения.

В следующей своей статье я расскажу Вам об Однородных дифференциальных уравнениях I-го порядка и о Линейных дифференциальных уравнениях I-го порядка, уравнении Бернулли.

Если у Вас есть желание более детально изучить данный материал, научиться решать задания по данным разделам, записывайтесь на мои занятия на сайте. Буду рад Вам помочь. Онлайн репетитор Андрей Зварыч.

© blog.tutoronline.ru, при полном или частичном копировании материала ссылка на первоисточник обязательна.

Понятие дифференциального уравнения

1.1Примеры моделей, приводящих к дифференциальным уравнениям

Прежде чем говорить о дифференциальных уравнения в общем виде, обсудим несколько простых примеров, в которых они возникают естественным образом.

1.1.1Рост населения. Мальтузианская модель

Пусть скорость роста популяции какого-нибудь вида (например, рыб в пруду или бактерий в чашке Петри) в любой момент времени пропорциональна количеству особей в популяции в этот момент времени. Это предположение кажется разумным (какая-то часть популяции за единицу времени воспроизводится), если есть достаточное количество ресурсов. Обозначим размер популяции в момент времени t через x(t). Тогда мгновенная скорость роста равна dx(t)dt. Обычно производная по переменной t обозначается точкой ˙x(t), а не штрихом. Таким образом, наш закон роста размера популяции можно записать так:

˙x(t)=kx(t),(1.1)

где k>0 — коэффициент пропорциональности (константа). Зависимость от t обычно опускают и пишут просто

˙x=kx.(1.2)

Это — одно из простейших (и важнейших) дифференциальных уравнений. Неизвестной величиной в ней является не число (как в обычных алгебраических уравнениях) и не вектор (как в линейной алгебре), а функция x(t).

1.1.2Рост экономики. Модель Солоу

Согласно модели Солоу, скорость прироста капиталовооруженности экономики (количества капитала в расчёте на одного трудоспособного человека) в предположении отсутствия внешней торговли, технического прогресса и роста населения, описывается формулой

˙k=sf(k)−δk,

где k=k(t) — капиталовооруженность экономики в момент времени t, s — норма сбережения, δ — норма выбытия капитала.

1.1.3Механическая система. Падающий шарик

Если я возьму в руку маленький тяжелый шарик, что с ним произойдёт, когда я его отпущу? Не нужно проводить этот эксперимент на практике и даже решать дифференциальное уравение, чтобы ответить: он станет падать вниз. Это подскажет нам наша физическая интуиция. Использование интуиции и ранее накопленного опыта очень важно при решении задач, поэтому мы время от времени будем обращаться к механическим примерам.

Пусть вертикальная координата шарика (высота) в момент времени t есть y(t). Известно, что на тело, находящееся в поле тяготения земли (на не слишком большой высоте) действует сила тяжести, равная

F=−mg,

где m — масса тела, g — ускорение свободного падения (примерно равно 9.8 м/с2), знак «-» выбран, поскольку сила тяжести действует в направлении «вниз» (против направления роста y). Трением мы будем пренебрегать и считать, что никаких других сил на шарик не действует.

Чтобы перейти к дифференциальным уравнениям, нужно вспомнить второй закон Ньютона, который гласит, что ускорение тела пропорционально действующей на него силе и обратно пропорционально массе:

a=F/m⇔F=ma.

Ускорение — это вторая производная от координаты по времени, она обозначается двумя точками. Таким образом, мы имеем дифференциальное уравнение, описывающее движение шарика:

¨y=−g.(1.3)

1.2Простейшие дифференциальные уравнения

Вернёмся к математической точке зрения на дифференциальные уравнения. Начнём с относительно общего определения.

1.2.1Дифференциальное уравнение общего вида

Дифференциальным уравнением называется соотношение вида

˙x=f(t,x),(1.4)

где x=x(t) — неизвестная функция, f(t,x) — известная функция двух переменных. Мы пока что будем рассматривать уравнения, в которых областью значений неизвестной функции являются вещественные числа R, но чуть позже обсудим и более сложные случаи, когда x принимает значение в многомерных пространствах. Также отметим, что в уравнении (1.4) фигурирует только первая производная неизвестной функции — это уравнение первого порядка. Позже мы обсудим, что делать с уравнениями более высоких порядков (например, таких как (1.3)). Пока же остановимся на рассмотрении уравнений вида (1.4).

Решением дифференциального уравнения называется дифференцируемая функция x=φ(t), такая, что при подстановке её в уравнение получается верное равенство:

˙φ(t)=f(t,φ(t))∀t∈D(f),(1.5)

где D(f) — область определения функции f: это может быть вся числовая ось, луч, отрезок, интервал или полуинтервал.

Рассмотрим несколько примеров.

1.2.2Нулевая правая часть

Простейшее дифференциальное уравнение, которое только можно придумать, имеет вид

˙x=0.

Его решениями являются функции x(t)=C, где C — любая константа. Действительно, если функция имеет нулевую производную и при этом всюду дифференцируема, то она не меняется и значит равна константе. Заметим, что даже в таком простейшем случае мы имеем не одно, а сразу целое семейство решений. Аналогичная ситуация будет и в более сложных примерах.

1.2.3Постоянная правая часть

Чуть более сложное уравнение:

˙x=k,

где k — константа. Это уравнение движения с постоянной скоростью, его решениями являются всевозможные линейные функции

x(t)=kt+C,

Заметим, что в этом случае константа C задаёт значение функции в начальный момент времени t=0.

1.2.4Правая часть, зависящая только от времени

Рассмотрим несколько более сложный пример: пусть функция f(t,x) в правой части (1.4) на самом деле не зависит от x.

˙x=f(t).(1.6)

Задачу отыскания решения такого дифференциального уравнения можно сформулировать следующим образом: для каждого значения независимой переменной t известна производная некоторой функции; найти эту функцию. Нетрудно видеть, что это в точности задача интегрирования (отыскания первообразной). Решение такого уравнения задаётся таким образом неопределенным интегралом, который можно записать в виде

x(t)=∫f(t)dt=∫tt0f(τ)dτ+C.(1.7)

Неопределенный интеграл по определению является семейством функций, а при записи его в виде определенного интеграла с переменным верхним пределом нужно указывать константу интегрирования явным образом.

1.2.5Начальные условия. Задача Коши

Чтобы выделить среди семейства решений дифференциального уравнения одно, обычно вместе с самим дифференциальным уравнением рассматривают дополнительное соотношение, называемое начальным условием — значение решения в какой-то момент времени (не обязательно t=0) полагают равным константе.

Когда задано дифференциальное уравнение и начальное условие, говорят, что поставлена задача Коши (по-английски Initial Value Problem). Например, можно рассмотреть такую задачу:

˙x=f(t),x(5)=0(1.8)

Eё решением будет уже только одна функция:

x(t)=∫t5f(τ)dτ(1.9)

Действительно, любой интеграл вида (1.7) является решением уравнения (1.6), а значит, и функция в (1.9) им является. Остаётся проверить начальное условие. При подстановке t=5 решение x(5)=∫55f(τ)dτ=0, то есть начальное условие выполняется.

Вопрос 1. Каким будет решение уравнения (1.6) при начальном условии x(5)=1?

  x(t)=∫15f(τ)dτ

Неверный ответ. Неверно, эта функция вообще является константой.

  x(t)=∫t5f(σ)dσ+1

Верный ответ. Верно!

  x(t)=∫ttf(τ)dτ+1

Неверный ответ. Неверно, обратите внимание на пределы интегрирования.

1.2.6Простейшее линейное уравнение

Положим в уравнении роста населения k=1. Получим следующее уравнение:

˙x=x(1.10)

Какие функции будут его решениями? Словами можно сказать, что условие, накладываемое этим уравнением, звучит так: «Производная функции равна самой этой функции». Одна известная функция обладает таким свойством — это экспонента x(t)=et. Нетрудно видеть, что если умножить экспоненту на любое число, получающаяся функция x(t)=Cet также будет решением этого уравнения. В частности, очевидно, что решением будет функция x(t)≡0.

Вопрос 2. Является ли решением уравнения (1.10) функция x(t)=et+C при C≠0?

  Да, при любых C≠0.

Неверный ответ. Это неверно, попробуйте подставить функцию в уравнение и посчитать производную.

  При некоторых C≠0 является, а при других нет.

Неверный ответ. Это неверно, попробуйте подставить функцию в уравнение и посчитать производную.

  Не является ни при каких C≠0.

Верный ответ. Верно, если подставить функцию в уравнение, C уничтожится при дифференцировании в левой части, но не уничтожится в правой. Таким образом, уравнение (1.10) принципиально отличается от уравнений вида (1.6), рассмотренных ранее.

1.3Геометрические объекты

В рассмотренных выше примерах неизвестная функция x(t) принимала значения во множестве вещественных чисел. В общем случае функция x(t) может принимать значения в других множествах — например, в многомерных пространствах. Множество, в котором принимает значение неизвестная функция (или, иными словами, множество всевозможных значений x(t) при каком-нибудь фиксированном t) называется фазовым пространством дифференциального уравнения. Множество точек вида (t,x) (декартово произведение фазового пространства на ось времени) называется расширенным фазовым пространством. График решения называется интегральной кривой. Интегральные кривые живут в расширенном фазовом пространстве. Построим некоторые интегральные кривые для уравнения ˙x=x. Как мы уже знаем, ими будут графики экспонент.

import matplotlib.pyplot as plt
import numpy as np
import qqmbr.odebook as ob
# see https://github.com/ischurov/qqmbr/blob/master/qqmbr/odebook.py
plt.rcParams['figure.figsize'] = (8, 6)
ob.axes4x4()
initials = list(range(-5, -1)) + [0.15] + [x/np.exp(1) for x in [1, 2, 3]]
initials.extend([-x for x in initials])
initials.append(0)
for C in initials:
    ob.mplot(np.linspace(-4,4),lambda t, C=C: C * np.exp(t),
             color='steelblue', linewidth=1.5) 

Рис. 1.1: Графики решений дифференциального уравнения ˙x=x

Если бы мы не знали, какие на самом деле решения нашего дифференциального уравнения (а это наиболее распространенный случай, чаще всего дифференциальные уравнения не решаются явно), мы всё равно могли бы примерно представить себе, как выглядят интегральные кривые. Чтобы это сделать, нам нужно построить поле направлений или поле прямых.

Вот что это такое. Возьмём произвольную точку P=(t0,x0) расширенного фазового пространства. Например, t0=1, x0=3. Мы можем провести в точке P касательную к интегральной кривой, проходящей через эту точку. Действительно, чтобы провести прямую через фиксированную точку, нужно знать только её угловой коэффициент, но угловой коэффициент касательной к графику некоторой функции равняется производной этой функции. А производную решения мы знаем, по определению решения она равна правой части уравнения. Для уравнения (1.10) правая часть в точке x равна x и, значит, касательная, проходящая через точку P, имеет угловой коэффициент, равный x0=3. Можно взять ещё несколько точек на прямой t=1 и провести соответствующие касательные через них. Получится такая картинка, см. рис. 1.2.

Рис. 1.2: Касательные к решениям

Вопрос 3. Почему прямые пересекаются в начале координат?

Понятно, что можно, действуя аналогично, построить касательные к решениям не только в выбранных точках, но и вообще в любой точке расширенного фазового пространства. В данном случае правая часть не зависит от t явно, поэтому через любые две точки, лежащие на одной горизонтальной прямой, будут проходить параллельные касательные. Мы будем рисовать только маленькие кусочки этих касательных.

Рис. 1.3: Поле направлений

На картинке изображены прямые, проходящие через какие-то конкретные точки, но на самом деле такая прямая может быть проведена через любую точку. Вся совокупность этих прямых и будет полем направлений.

Рис. 1.4: Поле направлений и интегральные кривые

Теперь задача отыскания решения дифференциального уравнения сводится к такой геометрической задаче: нужно найти кривую, которая в каждой своей точке касается прямой, принадлежащей полю направлений и проходящей через эту точку.

Эта интерпретация скоро окажется для нас очень полезной.

Дифференциальные уравнения используются для моделирования процессов, в которых участвует время. Предмет рассмотрения нашего курса — обыкновенные дифференциальные уравнения, они имеют вид ˙x=f(t,x), где x=x(t) — неизвестная функция, определённая на всей оси t или на какой-то его связной компоненте (отрезке, интервале, полуинтервале, луче). Решением дифференциального уравнения всегда является семейство функций; чтобы выбрать из них одну, нужно задать начальное условие. Множество всех возможных значений функции x называется фазовым пространством, а его декартово произведение на ось времени — расширенным фазовым пространством. График решения (кривая в расширенном фазовом пространстве) называется интегральной кривой. Если в каждой точке расширенного фазового пространства провести прямую, уголовой коэффициент которой равен значению правой части уравнения в этой точке, то получится поле прямых или поле направлений. Всякая интегральная кривая в каждой своей точке касается прямой из поля прямых, проходящей через данную точку.

Мы рассмотрели ряд примеров и ввели много новых понятий, но пока ничего не говорили о самом интригующем: как всё-таки решать дифференциальные уравнения? Короткий ответ неутешителен: дифференциальные уравнения обычно не решаются явно. (Если вас это расстраивает, подумайте о том, что обычные алгебраические уравнения начиная с пятой степени тоже как правило не решаются явно.) Тем не менее, мы научимся решать уравнения некоторых специальных классов (займёмся этим уже в следующей главе), а затем обсудим, что можно сделать с теми уравнениями, которые не решаются.


Следующая глава →

Заметки по R: Дифференциальные уравнения

library("knitr")
opts_chunk$set(
  # cache=FALSE,
               message=FALSE, warning=FALSE) 
library("ggplot2") # для построения графиков
library("rasterVis")
library("fields")
library("deSolve") # решение дифф. уравнений с начальными условиями
library("bvpSolve") # решение дифф. уравнений с краевыми условиями
library("dplyr") # манипуляции с данными

Пакет rasterVis предназначен для изображения данных на реальных географических картах, поэтому там нужно понятие проекции. Мы пока просто введем это шаманское заклинание

proj <- CRS('+proj=longlat +datum=WGS84')

Построим график векторного поля для системы:

\[ \left\{ \begin{array}{l} \dot{y}_1=y_2 \\ \dot{y}_2=y_1+\cos(y_2) \end{array} \right. 2), angle = atan2(y1dot, y2dot)) df2 <- df[c(«y1», «y2», «len», «angle»)] rast <- rasterFromXYZ(df2, crs = proj)

Строим классический график со стрелочками

vectorplot(rast, isField = TRUE)

Строим няку с капельками

streamplot(rast, isField = TRUE)

Простой график можно руками построить без доп. пакетов. При этом нам нужно самостоятельно уменьшить количество стрелочек.

y1 <- seq(-6, 6, 0.5)
y2 <- seq(-6, 6, 0.5)
df <- expand.grid(y1 = y1, y2 = y2)
df <- mutate(df, y1dot = y2, y2dot = y1 + cos(y2))
plot(df$y1, df$y2, pch = ".", xlab = expression(paste(y[1])),
  ylab = expression(paste(y[2])), main = "График векторного поля")
arrow.plot(df$y1, df$y2, df$y1dot, df$y2dot,
            arrow.ex = 0.03, length = 0.05)

Решим ОДУ с начальным условиями

Решим систему ОДУ с начальными условиями

Описываем саму систему:

eq1 <- function(t, y, parampampam) {
  return(list(c(
    y[2],
    y[1] + cos(y[2])    
  )))
}

Начальные условия:

y. start <- c(y1 = 1, y2 = 4) 

Точки, в которых компьютер будет считать функцию:

t <- seq(0, 10, by = 0.01)

Решаем

sol <- ode(y = y.start, times = t, func = eq1)
sol <- data.frame(sol)
head(sol)
##   time       y1       y2
## 1 0.00 1.000000 4.000000
## 2 0.01 1.040018 4.003678
## 3 0.02 1.080076 4.007785
## 4 0.03 1.120176 4.012326
## 5 0.04 1.160324 4.017305
## 6 0.05 1.200524 4.022725
str(sol)
## 'data.frame':    1001 obs. of  3 variables:
##  $ time: num  0 0.01 0.02 0.03 0.04 0.05 0.06 0.07 0.08 0.09 ...
##  $ y1  : num  1 1.04 1.08 1.12 1.16 ...
##  $ y2  : num  4 4 4.01 4.01 4.02 ...
ggplot(sol) + geom_line(aes(time, y1), size = 2) + labs(x = "t",
  y = expression(paste(y[1])), title = "Решение ОДУ с начальными условиями")

Функция ode возвращает матрицу, а для рисования графиков удобнее табличка с данными, data.frame. Строчка sol <- data.frame(sol) переделывает матрицу в таблицу с данными.

Решим систему ОДУ с краевыми условиями

Описываем саму систему:

eq1 <- function(t, y, parampampam) {
  return(list(c(
    y[2],
    y[1] + cos(y[2])    
  )))
}

Граничные условия:

y.start <- c(y1 = 1, y2 = NA) 
y.final <- c(y1 = 42, y2 = NA)

Точки, в которых компьютер будет считать функцию:

t <- seq(0, 10, by = 0.01)

Решаем

sol <- bvptwp(yini = y.start, yend = y.final,
           x = t, func = eq1,
           nmax = 2000)
sol <- data.frame(sol)
head(sol)
##      x        y1        y2
## 1 0.00 1.0000000 -1.553150
## 2 0.01 0.9845193 -1.543001
## 3 0.02 0.9691398 -1.532904
## 4 0.03 0.9538610 -1.522860
## 5 0.04 0.9386824 -1.512868
## 6 0.05 0.9236035 -1.502928
ggplot(sol) + geom_line(aes(x, y1), size = 2) +  labs(x = "x", 
  y = expression(paste(y[1])), title = "Решение ОДУ с краевыми условиями")

Бесплатное приложение. Изображение функций двух переменных

Есть несколько способов представить себе функцию от двух переменных, \(z(x, y)\):

  • 3D график
  • Линии уровня
  • Векторное поле градиентов функции

Создаем data. 2))

r <- rasterFromXYZ(df, crs = proj)

Линии уровня функции z

contour(r)

Капельки текущие по градиенту

streamplot(r)

Направление градиентов, заодно вид сбоку для графика функции

vectorplot(r)

Решение линейных дифференциальных уравнений первого порядка

Вы могли бы сначала прочитать о дифференциальных уравнениях
и о разделении переменных!

Дифференциальное уравнение – это уравнение с функцией и одной или несколькими ее производными:


Пример: уравнение с функцией y и ее производная д дх  

Здесь мы рассмотрим решение специального класса дифференциальных уравнений под названием 9.0009 Линейные дифференциальные уравнения первого порядка

Первый заказ

Они «Первый Орден», когда есть только д дх , а не г 2 г дх 2 или г 3 г дх 3 и т. д.

Линейный

Дифференциальное уравнение первого порядка является линейным , когда это можно сделать так:

д дх + Р(х)у = Q(х)

Где P(x) и Q(x) являются функциями x.

Для ее решения есть специальный метод:

  • Мы изобрели две новые функции от x, назовем их u и v и скажем, что y=uv .
  • Затем мы решаем найти u , а затем найти v , привести в порядок и готово!

И мы также используем производную от y=uv (см. Производные правила (правило произведения)):

д дх = ты дв дх + в дю дх

шагов

Вот пошаговый метод их решения:

Попробуем посмотреть пример:

Пример 1: Решите это:

д дх г х = 1

Во-первых, линейно ли это? Да, как есть в форме

д дх + P(x)y = Q(x)
, где P(x) = − 1 х и Q(x) = 1

Итак, давайте выполним шаги:

Шаг 1: Замените y = uv и  . д дх = ты дв дх + в дю дх

Итак: д дх г х = 1

Становится следующим:u дв дх + в дю дх уф х = 1

Шаг 2: Фактор частей, включающих v

Фактор v :u дв дх + v( дю дх и х ) = 1

Шаг 3: Приравняем член v к нулю

v член приравняем к нулю: дю дх и х = 0

Итак: дю дх знак равно и x

Шаг 4: Решите, используя разделение переменных, чтобы найти u

Отдельные переменные: дю и знак равно дх x

Поставьте знак интеграла: ∫ дю и = ∫ дх x

Интегрируем: ln(u) = ln(x) + C

Сделать C = ln(k):ln(u) = ln(x) + ln(k)

Итак:u = kx

Шаг 5: Подставить u обратно в уравнение на шаге 2

(Помните, что термин v равен 0, поэтому его можно игнорировать): kx дв дх = 1

Шаг 6: Решите это, чтобы найти v

Отдельные переменные: k dv = дх x

Поставьте знак интеграла: ∫k dv = ∫ дх x

Интегрируем: kv = ln(x) + C

Делаем C = ln(c):kv = ln(x) + ln(c)

Итак: kv = ln(cx)

И Итак: v = 1 к ln(cx)

Шаг 7: Подставьте в y = uv , чтобы найти решение исходного уравнения.

у = ув:у = кх 1 к ln(cx)

Упрощение: y = x ln(cx)

И это дает это прекрасное семейство кривых:


y = x ln(cx) для различных значений c

Что означают эти кривые?

Они являются решением уравнения   д дх г х = 1

Другими словами:

В любом месте на любой из этих кривых
наклон минус г х равно 1

Давайте проверим несколько точек на с=0,6 кривая:

Оценка вне графика (до 1 знака после запятой):

Точка х и Уклон ( д дх ) д дх г х
А 0,6 −0,6 0 0 — −0,6 0,6 = 0 + 1 = 1
Б 1,6 0 1 1 — 0 1,6 = 1 — 0 = 1
С 2,5 1 1,4 1,4 — 1 2,5 = 1,4 — 0,4 = 1

Почему бы не проверить несколько точек самостоятельно? Вы можете построить кривую здесь.

 

Возможно, вам поможет еще один пример? Может чуть сложнее?

Пример 2: Решите это:

д дх 3 года х = х

Во-первых, это линейно? Да, как есть в форме

д дх + P(x)y = Q(x)
, где P(x) = − 3 х и Q(x) = x

Итак, давайте выполним шаги:

Шаг 1: Замените y = uv и  . д дх = ты дв дх + в дю дх

Итак: д дх 3 года х = x

Становится следующим: u дв дх + в дю дх 3уф х = x

Шаг 2: Фактор частей, включающих v

Фактор v дв дх + v( дю дх х ) = x

Шаг 3: Положите член v равным нулю

v член = ноль: дю дх х = 0

Итак: дю дх знак равно x

Шаг 4: Решите, используя разделение переменных, чтобы найти u

Отдельные переменные: дю и = 3 дх x

Поставьте знак интеграла: ∫ дю и = 3 ∫ дх x

Интегрируем: ln(u) = 3 ln(x) + C

Делаем C = −ln(k):ln(u) + ln(k) = 3ln(x)

Тогда:uk = x 3

Итак:u = х 3 k

Шаг 5: Подставьте u обратно в уравнение на шаге 2

(помните, что член v равен 0, поэтому его можно игнорировать):( х 3 к ) дв дх = x

Шаг 6: Решите это, чтобы найти v

Отдельные переменные: dv = k x -2 dx

Поставьте знак интеграла: ∫dv = ∫k x -2 dx

Интегрируйте: v = −k x -1 + D

Шаг 7: Подставьте 0y, чтобы найти 90uv 09 09 решение исходного уравнения.

у = ув: у = х 3 к ( −k x -1 + D )

Упрощение: y = −x 2 + Д к х 3

Заменить D/k с одной константой c : y = с x 3 − x 2

И это дает это прекрасное семейство кривых:


y = c x 3 − x 2 для различных значений c

И еще один пример, на этот раз еще сложнее :

Пример 3: Решите это:

д дх + 2xy= −2x 3

Во-первых, линейно ли это? Да, как это в форме

д дх + P(x)y = Q(x)
, где P(x) = 2x и Q(x) = −2x 3

Итак, давайте выполним шаги:

Шаг 1: Замените y = uv и  . д дх = ты дв дх + в дю дх

Итак: д дх + 2xy= −2x 3

Получается так: u дв дх + в дю дх + 2xув = −2x 3

Шаг 2: Фактор частей, включающих v

Фактор v :u дв дх + v( дю дх + 2xu ) = -2x 3

Шаг 3: Приравняем терм v к нулю

v term = ноль: дю дх + 2xu = 0

Шаг 4: Решите, используя разделение переменных, чтобы найти u

Отдельные переменные: дю и = −2x dx

Поставьте знак интеграла: ∫ дю и = −2∫x dx

Интегрируем: ln(u) = −x 2 + C

Сделать C = −ln(k):ln(u) + ln(k) = −x 2

Тогда: ты = е 2 k

Шаг 5: Подставьте u обратно в уравнение на шаге 2

(помните, что член v равен 0, поэтому его можно игнорировать):( е 2 к ) дв дх = −2x 3

Шаг 6: Решите это, чтобы найти v

Отдельные переменные: dv = −2k x 3 e x 2 dx

4 интегральное число: = ∫−2k x 3 e x 2 dx

Интегрируем: v = о нет! это трудно!

Давайте посмотрим. .. мы можем интегрировать по частям… что говорит:

∫RS dx = R∫S dx − ∫R’ ( ∫S dx) dx

(Примечание: мы используем R и S здесь использование u и v может сбивать с толку, поскольку они уже означают что-то другое.)

Выбор R и S очень важен, это лучший выбор, который мы нашли:

  • R = −x 2 и
  • S = 2x e x 2

Итак, вперед:

Сначала вытащите k:v = k∫−2x 3 e x 2 dx

R = -x 2 и S = 2x e x 4 : 2 2 = k∫(−x 2 )(2xe x 2 ) dx

Теперь интегрируем по частям:v = kR∫S dx − k∫R’ ( ∫ S dx) dx

Подставим R = −x 2 и S = ​​2x e x 2

А также R’ = −2x и ∫ S dx = e x 2

Таким образом, получается: v = −kx 2 ∫2x e x 2 dx − k∫−2x (e x 2 ) dx

Теперь интегрируем:v = −kx 2 e x 2 + k e x 2 + D

Упростить:v = ke x 2 (1−x 2 ) + D

Шаг 7: Подставьте в y = uv , чтобы найти решение исходного уравнения.

у = ув: у = е 2 к ( ke x 2 (1−x 2 ) + D )

Упрощение: y =1 − x 2 + ( Д к )e x 2

Замените D/k одной константой c : y = 1 − x 2 + с e x 2

И мы получаем это прекрасное семейство кривых:


y = 1 − x 2 + с e x 2 для различных значений c

 

9429, 9430, 9431, 9432, 9433, 9434, 9435, 9436, 9437, 9438

1. Решение дифференциальных уравнений

Дифференциальное уравнение (или «DE») содержит производные или дифференциалы .

Наша задача решить дифференциальное уравнение. В какой-то момент это потребует интегрирования, и мы (в основном) получим выражение типа « y = …».

Вспомним из раздела «Дифференциал» главы «Интеграция», что дифференциал можно рассматривать как , производная от , где dy/dx на самом деле не записывается в форме дроби.

Примеры дифференциалов

На этой странице…

Определения порядка и степени
Общие и частные решения
ДУ второго порядка

dx (это означает «бесконечно малое изменение в x «)

`d\theta` (это означает «бесконечно малое изменение `\theta`»)

`dt` (это означает «бесконечно малое изменение в 93/3-3х+К`

Но куда делись dy из `(dy)/(dx)`? Почему казалось, что оно исчезло?

В этом примере кажется, что мы интегрируем только часть x (справа), но на самом деле мы интегрировали также и по отношению к y (слева). DE такие — вам нужно интегрировать по двум (иногда больше) различным переменным, по одной за раз. 2 — 3)dx` 93/3-3х+К`

В левой части мы интегрировали `int dy = int 1 dy`, чтобы получить лет.

Примечание о константе: Мы проинтегрировали обе стороны, но константа интегрирования есть только с правой стороны. Что случилось с тем, что слева? Ответ довольно прост. На самом деле мы получаем константу с обеих сторон, но мы можем объединить их в одну константу ( K ), которую запишем в правой части.

93}/3 = -cos(t + 0,2) + K`

Проинтегрировали по θ слева и по t справа.

Вот график нашего решения для `K=2`:

Типичный график решения для дифференциального уравнения: theta(t)=root(3)(-3cos(t+0.2)+6)π2π3π−π123tθОткрыть изображение на новой странице

Типичный график решения для Примера 2 DE: `theta(t )=корень(3)(-3cos(t+0,2)+6)`.

Решение дифференциального уравнения

Из приведенных выше примеров видно, что решить DE означает найти уравнение без производных, удовлетворяющее заданному DE. Решение дифференциального уравнения всегда требует одного или нескольких интеграции шагов.

Важно уметь определять тип DE , с которым мы имеем дело, прежде чем пытаться реши.

Определения

Первый порядок DE: Содержит только первые производные

Второй порядок DE: Содержит вторые производные (и возможно также первые производные) 97-5y=3`

Это DE имеет порядка 2 (наибольшая производная — это вторая производная ) и степень 4 (степень старшей производной равно 4.)

Общие и частные решения

Когда мы впервые выполнили интеграцию, мы получили общий раствор (с константой K ).

Мы получили частное решение путем подстановки известного значения для х и и . Эти известные условия называется граничными условиями (или начальными условия ).

То же самое и при решении дифференциальных уравнений: сначала найдите общее решение, а затем подставьте заданные числа, чтобы найти частные решения.

Давайте рассмотрим несколько примеров ДУ первого порядка и первой степени.

Пример 4

а. Найдите общее решение для дифференциала уравнение

92+K`

Ответ тот же — способ его написания и осмысления немного отличается.


ПРИМЕЧАНИЕ 2: `int dy` означает `int1 dy`, что дает нам ответ `y`.

Мы также могли бы:

`intdt=t`

`intd тета=тета`

`интервал да=а`

и так далее. В этом разделе мы будем часто встречаться с такими интегралами.

(b) Теперь мы используем информацию y (0) = 3, чтобы найти 92 + 3`.

Пример 5

Найдите частное решение

`у’ = 5`

, учитывая, что когда `x=0, y=2`.

Ответить

Мы можем написать

у’ = 5

как дифференциальное уравнение:

дх = 5 дх

Интеграция обеих сторон дает:

у = 5 х + К

Применяя граничные условия: x = 0, y = 2, имеем K = 2, значит:

у = 5 х + 2

Пример 6

Найдите частное решение

`у»’ = 0`

при том, что:

`у(0) = 3, «у'(1) = 4, «у»(2) = 6`

Ответить

Поскольку y»’ = 0, когда мы интегрируем один раз, мы получаем: 92)/2 + Bx + C` .

Сейчас

y (0) = 3 дает C = 3.

и

(2) = 6 дает A = 6

(На самом деле, = 6 для любого значения x в этой задаче, поскольку члена x нет)

Наконец,

y’ (1) = 4 дает B = -2.

Итак, конкретное решение этого вопроса:

у = 3 х 2 2 х + 3

Проверка решения дифференцированием и подстановкой начальных условий:

у’ = 6 х 2

y’ (1) = 6(1) 2 = 4

у» = 6

у»’ = 0

Наше решение верное.

Пример 7

После решения дифференциала уравнение,

`(dy)/(dx)ln x-y/x=0`

(мы увидим, как решать это ДУ в следующем раздел Разделение переменных) получаем результат

`y=c ln x`

Мы получили правильное общее решение?

Ответить

Теперь, если `y=c ln x`, то `(dy)/(dx)=c/x`

[См. Производная логарифмической функции, если вы заржавели в этом.)

Так

`»LHS»=(dy)/(dx)ln x-y/x`

`=(c/x) ln x-((c ln x))/x`

`=0`

`=»RHS»`

Делаем вывод, что у нас есть правильное решение.

DE второго порядка

Мы включили сюда еще два примера, чтобы дать вам представление о DE второго порядка. Позже в этой главе мы увидим, как решать такие линейные дифференциальные уравнения второго порядка.

Пример 8

Общий раствор второго порядка ДЭ

у » + а 2 у = 0

это

`y = A cos ax + B sin ax`

Пример 9

Общий раствор второго порядка ДЭ

у » — 3 у ‘ + 2 у = 0

это

y = Ae 2 x + Be x

Если у нас есть следующие граничные условия:

у (0) = 4, у’ (0) = 5

, то конкретное решение определяется как:

y = e 2 x + 3 e x


Теперь мы делаем несколько примеров с использованием DE второго порядка, где нам дается окончательный ответ, и нам нужно проверить, является ли это правильным решением.

Пример 10 — второй порядок DE 92)+4y=0`

Ответить

У нас есть дифференциальное уравнение второго порядка, и нам дано общее решение. Наша задача показать правильность решения.

Мы делаем это, подставляя ответ в исходное дифференциальное уравнение 2-го порядка.

Нам нужно найти вторую производную от y :

y = c 1 sin 2 x + 3 cos 2 x

92)=2(dy)/(dx)`

Нужна помощь в решении другой задачи исчисления? Попробуйте решение проблем.

Отказ от ответственности: IntMath.com не гарантирует точность результатов. Решатель задач предоставлен Mathway.

Дифференциальные уравнения — Линейные уравнения

Онлайн-заметки Пола
Главная / Дифференциальные уравнения / DE первого порядка / Линейные уравнения

Показать мобильное уведомление Показать все примечания Скрыть все примечания

Уведомление для мобильных устройств

Похоже, вы используете устройство с «узкой» шириной экрана ( т. е. вы наверное на мобильном телефоне). Из-за характера математики на этом сайте лучше всего просматривать в ландшафтном режиме. Если ваше устройство не находится в ландшафтном режиме, многие уравнения будут отображаться сбоку вашего устройства (должна быть возможность прокрутки, чтобы увидеть их), а некоторые пункты меню будут обрезаны из-за узкой ширины экрана.

Раздел 2-1: Линейные дифференциальные уравнения

Первый частный случай дифференциальных уравнений первого порядка, который мы рассмотрим, — это линейные дифференциальные уравнения первого порядка. В этом случае, в отличие от большинства случаев первого порядка, которые мы рассмотрим, мы действительно можем вывести формулу для общего решения. Общее решение выводится ниже. Однако мы бы посоветовали вам не запоминать саму формулу. Вместо того, чтобы запоминать формулу, вы должны запомнить и понять процесс, который я собираюсь использовать для вывода формулы. На самом деле с большинством задач проще работать, используя процесс, а не формулу.

Итак, давайте посмотрим, как решить линейное дифференциальное уравнение первого порядка. Помните, что по мере прохождения этого процесса цель состоит в том, чтобы прийти к решению в форме \(y = y\left( t \right)\). Иногда легко упустить цель из виду, когда мы впервые проходим через этот процесс.

Чтобы решить линейное дифференциальное уравнение первого порядка, мы ДОЛЖНЫ начать с дифференциального уравнения в форме, показанной ниже. Если дифференциальное уравнение не в такой форме, то процесс, который мы собираемся использовать, не будет работать.

\[\begin{equation}\frac{{dy}}{{dt}} + p\left( t \right)y = g\left( t \right) \label{eq:eq1} \end{equation} \]

Где обе \(p(t)\) и \(g(t)\) являются непрерывными функциями. Напомним, что быстрое и грязное определение непрерывной функции состоит в том, что функция будет непрерывной при условии, что вы можете рисовать график слева направо, даже не поднимая карандаш/ручку. Другими словами, функция непрерывна, если в ней нет дыр и разрывов.

Теперь предположим, что где-то в мире существует некая волшебная функция \(\mu \left( t \right)\), называемая интегрирующим множителем . На данный момент не беспокойтесь о том, что это за функция или откуда она взялась. Мы выясним, что такое \(\mu \left( t \right)\), когда получим формулу общего решения.

Итак, теперь, когда мы предположили существование \(\mu \left( t \right)\), умножьте все в \(\eqref{eq:eq1}\) на \(\mu \left( t \right) )\). Это даст.

\[\begin{equation}\mu \left( t \right)\frac{{dy}}{{dt}} + \mu \left( t \right)p\left( t \right)y = \mu \left( t \right)g\left( t \right) \label{eq:eq2} \end{equation}\]

Теперь в игру вступает магия \(\mu \left( t \right)\). Мы собираемся предположить, что чем бы ни было \(\mu \left( t \right)\), оно удовлетворяет следующему.

\[\begin{equation}\mu \left( t \right)p\left( t \right) = \mu ‘\left( t \right) \label{eq:eq3} \end{equation}\]

Снова не беспокойтесь о том, как мы можем найти \(\mu \left( t \right)\), который будет удовлетворять \(\eqref{eq:eq3}\). Как мы увидим, если \(p(t)\) непрерывно, мы можем его найти. Итак, заменив \(\eqref{eq:eq3}\), мы теперь приходим к.

\[\begin{equation}\mu \left( t \right)\frac{{dy}}{{dt}} + \mu ‘\left( t \right)y = \mu \left( t \right) g\left( t \right) \label{eq:eq4} \end{equation}\]

На данный момент мы должны признать, что левая часть \(\eqref{eq:eq4}\) есть не что иное, как следующее правило произведения. 9\prime}\,dt}} = \int{{\mu \left( t \right)g\left( t \right)\,dt}}\] \[\begin{equation}\mu \left( t \right)y\left( t \right) + c = \int{{\mu \left( t \right)g\left( t \right)\, dt}} \label{eq:eq6} \end{уравнение}\]

Обратите внимание, что сюда включена постоянная интегрирования \(c\) из левой части интегрирования. Очень важно, чтобы это было включено. Если его не указать, вы каждый раз будете получать неправильный ответ.

Последним шагом является алгебраическое решение для решения \(y(t)\).

\[\ begin{align*}\mu \left(t\right)y\left(t\right) & = \int{{\mu \left(t\right)g\left(t\right)\, dt}} — c\\ y\left( t \right) & = \frac{{\int{{\mu \left( t \right)g\left( t \right)\,dt}} — c} }{{\ mu \left( t \right)}}\end{align*}\]

Теперь, с точки зрения записи, мы знаем, что постоянная интегрирования \(c\) является неизвестной константой, и поэтому, чтобы упростить нашу жизнь, мы включим знак минус перед ней в константу и используем плюс вместо. Это НЕ повлияет на окончательный ответ для решения. Итак, с этим изменением мы имеем.

\[\ begin{equation}y\left(t\right) = \frac{{\int{{\mu \left(t\right)g\left(t\right)\,dt}} + c}} {{\mu \left( t \right)}} \label{eq:eq7} \end{equation}\]

Опять же, смена знака у константы не повлияет на наш ответ. Если вы решите сохранить знак минус, вы получите то же значение \(c\), что и мы, за исключением того, что оно будет иметь противоположный знак. При подключении \(c\) мы получим точно такой же ответ.

В этом разделе много быстрых и лузовых игр с константами интегрирования, так что вам нужно будет к этому привыкнуть. Когда мы делаем это, мы всегда стараемся четко объяснить, что происходит, и пытаемся оправдать, почему мы сделали то, что сделали.

Итак, теперь, когда у нас есть общее решение \(\eqref{eq:eq1}\), нам нужно вернуться и определить, что это за магическая функция \(\mu \left( t \right)\) является. На самом деле это более простой процесс, чем вы думаете. Начнем с \(\eqref{eq:eq3}\).

\[\mu \left( t \right)p\left( t \right) = \mu ‘\left( t \right)\]

Разделить обе части на \(\mu \left( t \right)\),

\[\ frac{{\mu ‘\left( t \right)}}{{\mu \left( t \right)}} = p\left( t \right)\] 9\prime} = p\left( t \right)\]

Как и в случае с процессом, все, что нам нужно сделать, это объединить обе стороны, чтобы получить.

\[\begin{align*}\ln \mu \left( t \right) + k &= \int{{p\left( t \right)\,dt}}\\ \ln \mu \left( t \right) & = \int{{p\left( t \right)\,dt}} + k\end{align*}\]

Вы заметите, что постоянная интегрирования из левой части, \(k\), была перемещена в правую часть, и в нее снова был включен знак минус, как мы делали ранее. Также обратите внимание, что мы используем \(k\) здесь, потому что мы уже использовали \(c\), и через некоторое время мы будем иметь их оба в одном уравнении. Итак, чтобы избежать путаницы, мы использовали разные буквы для обозначения того факта, что они, по всей вероятности, будут иметь разные значения. 9{\ int {{p \ left ( t \ right) \, dt}}}} \ label {eq: eq8} \ end {equation} \]

Итак, теперь у нас есть формула для общего решения, \(\eqref{eq:eq7}\), и формула для интегрирующего коэффициента, \(\eqref{eq:eq8}\). Однако у нас есть проблема. У нас есть две неизвестные константы, и чем больше у нас неизвестных констант, тем больше проблем у нас будет позже. Поэтому было бы неплохо, если бы мы могли найти способ устранить один из них (не будем быть в состоянии устранить оба….). 9{\int{{p\left(t\right)\,dt}}}}}}\end{align*}\]

Таким образом, \(\eqref{eq:eq7}\) можно записать таким образом, что единственное место, где появляются две неизвестные константы, — это их отношение. Тогда, поскольку и \(c\), и \(k\) являются неизвестными константами, отношение этих двух констант равно. Поэтому мы просто назовем соотношение \(c\), а затем исключим \(k\) из \(\eqref{eq:eq8}\), так как в конце концов оно будет поглощено \(c\).

Тогда решение линейного дифференциального уравнения первого порядка будет 9{\ int {{p \ left ( t \ right) \, dt}}}} \ label {eq: eq10} \ end {equation} \]

Реальность такова, что \(\eqref{eq:eq9}\) не так полезен, как может показаться. Часто бывает проще просто запустить процесс, который привел нас к \(\eqref{eq:eq9}\), чем использовать формулу. Мы не будем использовать эту формулу ни в одном из наших примеров. Нам нужно будет регулярно использовать \(\eqref{eq:eq10}\), так как эту формулу проще использовать, чем процесс ее получения.

Процесс решения

Процесс решения линейного дифференциального уравнения первого порядка выглядит следующим образом.

  1. Приведите дифференциальное уравнение к правильной начальной форме \(\eqref{eq:eq1}\).
  2. Найдите коэффициент интегрирования, \(\mu \left( t \right)\), используя \(\eqref{eq:eq10}\).
  3. Умножьте все в дифференциальном уравнении на \(\mu \left( t \right)\) и убедитесь, что левая часть становится правилом произведения \(\left( {\mu \left( t \right)y\left( t \right)} \right)’\) и напишите так.
  4. Интегрируйте обе стороны, убедитесь, что вы правильно работаете с константой интегрирования.
  5. Найдите решение \(y(t)\).

Давайте рассмотрим пару примеров. Начнем с решения дифференциального уравнения, которое мы вывели в разделе «Поле направлений».

Пример 1 Найдите решение следующего дифференциального уравнения. \[\frac{{dv}}{{dt}} = 9,8 — 0,196v\]

Показать решение

Во-первых, нам нужно получить дифференциальное уравнение в правильной форме. 9{ — 0,196т}}\]

Из решения этого примера мы теперь можем понять, почему постоянная интегрирования так важна в этом процессе. Без него в этом случае мы получили бы единственное постоянное решение \(v(t)=50\). С константой интегрирования мы получаем бесконечно много решений, по одному для каждого значения \(с\).

Вернувшись к разделу поля направления, где мы впервые вывели дифференциальное уравнение, используемое в последнем примере, мы использовали поле направления, чтобы набросать некоторые решения. Посмотрим, правильно ли мы их поняли. Все, что нам нужно сделать, чтобы набросать некоторые решения, это выбрать различные значения \(c\), чтобы получить решение. Некоторые из них показаны на графике ниже.

Итак, похоже, мы неплохо нарисовали графики в разделе поля направления.

Теперь вспомните из раздела «Определения», что начальные условия позволяют нам сосредоточиться на конкретном решении. Решения дифференциальных уравнений первого порядка (не только линейных, как мы увидим) будут содержать единственную неизвестную константу, поэтому нам потребуется ровно одно начальное условие, чтобы найти значение этой константы и, следовательно, найти решение, которое мы искали. Начальное условие для дифференциальных уравнений первого порядка будет иметь вид

\[y\влево( {{t_0}} \вправо) = {y_0}\]

Напомним также, что дифференциальное уравнение вместе с достаточным количеством начальных условий называется начальной задачей (IVP).

Пример 2. Решите следующую IVP. \[\frac{{dv}}{{dt}} = 9,8 — 0,196v\hspace{0,25 дюйма}v\left( 0 \right) = 48\]

Показать решение

Чтобы найти решение IVP, мы должны сначала найти общее решение дифференциального уравнения, а затем использовать начальное условие, чтобы определить точное решение, которое нам нужно. Итак, поскольку это то же самое дифференциальное уравнение, которое мы рассматривали в примере 1, 9{ — 0,196т}}\]

Теперь, чтобы найти решение, которое мы ищем, нам нужно определить значение \(c\), которое даст нам решение, которое мы ищем. Для этого мы просто подставляем начальное условие, которое дает нам уравнение, которое мы можем решить для \(c\). Итак, давайте сделаем это

\[48 = v\влево( 0 \вправо) = 50 + c\hspace{0,25 дюйма} \Стрелка вправо \hspace{0,25 дюйма}c = — 2\]

Итак, актуальное решение IVP. 9{\ пер \, \, \ сек \ влево ( х \ вправо)}} = \ сек \ влево ( х \ вправо) \]

Вы можете сделать интеграл? Если нет, перепишите тангенс обратно в синусы и косинусы, а затем используйте простую замену. Обратите внимание, что мы можем опустить столбцы абсолютных значений секанса из-за ограничений на \(x\). Собственно, в этом и причина ограничений на \(x\). Отметим также, что есть две формы ответа на этот интеграл. Они эквивалентны, как показано ниже. То, что вы используете, действительно вопрос предпочтений. 9{\ln f\left(x\right)}} = f\left(x\right) \label{eq:eq11}\end{equation}\]

Это важный факт, который вы всегда должны помнить при возникновении подобных проблем. Мы хотим максимально упростить интегрирующий фактор во всех случаях, и этот факт поможет в этом упрощении.

Вернемся к примеру. Умножьте интегрирующий коэффициент через дифференциальное уравнение и убедитесь, что левая часть является правилом произведения. Также обратите внимание, что мы умножаем интегрирующий коэффициент на переписанное дифференциальное уравнение, а НЕ на исходное дифференциальное уравнение. Убедитесь, что вы делаете это. Если вы умножите интегрирующий коэффициент на исходное дифференциальное уравнение, вы получите неправильное решение! 92}\left( x \right)\,dx}}\\ \sec \left( x \right)y\left( x \right) & = — \frac{1}{2}\cos \left( { 2x} \right) — \tan \left( x \right) + c\end{align*}\]

Обратите внимание на использование тригонометрической формулы \(\sin \left( {2\theta } \right) = 2\sin \theta \cos \theta \), которая упростила вычисление интеграла. Далее ищите решение.

\[\begin{align*}y\left( x \right) & = — \frac{1}{2}\cos \left( x \right)\cos \left( {2x} \right) — \cos \left( x \right)\tan \left( x \right) + c\cos \left( x \right)\\ & = — \frac{1}{2}\cos \left( x \right)\ cos \left( {2x} \right) — \sin \left( x \right) + c\cos \left( x \right)\end{align*}\]

Наконец, примените начальное условие, чтобы найти значение \(c\).

\[\ begin{align*}3\sqrt 2 = y\left( {\frac{\pi} {4}} \right) & = — \frac{1}{2}\cos \left({\frac {\pi} {4}} \right)\cos \left( {\frac{\pi} {2}} \right) — \sin \left({\frac{\pi} {4}} \right) + c \ cos \ left ( {\ frac {\ pi} {4}} \ right) \\ 3 \ sqrt 2 & = — \ frac {{\ sqrt 2}} {2} + c \ frac {{\ sqrt 2 }}{2}\\ c & = 7\end{align*}\]

Решение есть. 92} — t + 1\hspace{0.25in}y\left( 1 \right) = \frac{1}{2}\]

Показать решение

Сначала разделите на t, чтобы получить дифференциальное уравнение в правильной форме. 2} + t\] 92}\]

Ниже приведен график решения.

Давайте поработаем над последним примером, в котором больше рассматривается интерпретация решения, чем поиск решения.

Пример 6. Найдите решение следующей IVP и определите все возможные варианты поведения решения как \(t \to \infty \). Если это поведение зависит от значения \(y_{0}\), укажите эту зависимость. \[2y’ — y = 4\sin \left( {3t} \right)\hspace{0.25in}y\left(0 \right) = {y_0}\] 9{\ гидроразрыва {т} {2}}} \ конец {выровнять *} \]

Примените начальное условие, чтобы найти значение \(c\) и обратите внимание, что оно будет содержать \(y_{0}\), так как у нас нет значения для этого.

\[{y_0} = y\left( 0 \right) = — \frac{{24}}{{37}} + c\hspace{0,25 дюйма}\,\,\,\, \Rightarrow \hspace{0,25 in}\,\,c = {y_0} + \frac{{24}}{{37}}\]

Итак, решение

\[y\left( t \right) = — \frac{{24}}{{37}}\cos \left( {3t} \right)\, — \frac{4}{{37}}\sin \left( {3t} \right)\, + \left( {{y_0} + \frac{{24}}{{37}}} \right){{\bf{e}}^{\frac{t {2}}}\]

Теперь, когда у нас есть решение, давайте посмотрим на долгосрочное поведение ( т. е. \(t \to \infty \)) решения. Первые два члена решения останутся конечными при всех значениях \(t\). Именно последнее слагаемое будет определять поведение решения. Экспонента всегда будет стремиться к бесконечности как \(t \to \infty\), однако в зависимости от знака коэффициента \(c\) (да, мы уже нашли это, но для простоты этого обсуждения мы продолжим назвать его \(с\)). В следующей таблице показано долгосрочное поведение решения для всех значений \(c\).

Диапазон \(с\) Поведение решения как\(t\to\infty\)
\(с\) < 0 \(у\влево(т\вправо) \к — \infty\)
\(с\) = 0 \(y\left( t \right)\) остается конечным
\(с\) > 0 \(у\влево(т\вправо) \в\infty\)

Такое поведение также можно увидеть на следующем графике нескольких решений.

Теперь, поскольку мы знаем, как \(c\) связано с \(y_{0}\), мы можем связать поведение решения с \(y_{0}\). В следующей таблице показано поведение решения с точки зрения \(y_{0}\) вместо \(c\).

Диапазон \(y_{0}\)

Поведение решения как\(t\to\infty\)

\({y_0} < - \frac{{24}}{{37}}\) \(у\влево(т\вправо) \к — \infty\)
\({y_0} = — \frac{{24}}{{37}}\) \(y\left( t \right)\) остается конечным
\({y_0} > — \frac{{24}}{{37}}\) \(у\влево(т\вправо) \в\infty\)

Обратите внимание, что при \({y_0} = — \frac{{24}}{{37}}\) решение останется конечным. Это не всегда будет происходить.

Изучение долгосрочного поведения решений иногда важнее, чем само решение. Предположим, что приведенное выше решение дало температуру в металлическом стержне. В этом случае мы хотели бы, чтобы решение (решения) оставалось конечным в долгосрочной перспективе. Благодаря этому исследованию у нас теперь будет значение начального условия, которое даст нам это решение, и, что более важно, значения начального условия, которых нам нужно избегать, чтобы не расплавить брусок.

Дифференциальные уравнения

Показать мобильное уведомление Показать все примечания Скрыть все примечания

Уведомление для мобильных устройств

Похоже, вы используете устройство с «узкой» шириной экрана ( т.е. вы наверное на мобильном телефоне). Из-за характера математики на этом сайте лучше всего просматривать в ландшафтном режиме. Если ваше устройство не находится в ландшафтном режиме, многие уравнения будут отображаться сбоку вашего устройства (должна быть возможность прокрутки, чтобы увидеть их), а некоторые пункты меню будут обрезаны из-за узкой ширины экрана.

Вот мои заметки к курсу дифференциальных уравнений, который я преподаю здесь, в Ламарском университете. Несмотря на то, что это мои «заметки о занятиях», они должны быть доступны всем, кто хочет научиться решать дифференциальные уравнения или нуждается в повторении дифференциальных уравнений.
Я пытался сделать эти заметки как можно более автономными, поэтому вся информация, необходимая для их прочтения, взята либо из уроков исчисления или алгебры, либо содержится в других разделах заметок.
Вот пара предупреждений для моих студентов, которые могут быть здесь, чтобы получить копию того, что произошло в день, который вы пропустили.

  1. Поскольку я хотел сделать это довольно полным набором заметок для всех, кто хочет изучать дифференциальные уравнения, я включил некоторые материалы, которые у меня обычно не хватает времени для изучения в классе, и поскольку они меняются от семестра к семестру, они здесь не упоминаются. . Вам нужно будет найти одного из своих одноклассников, чтобы узнать, есть ли в этих заметках что-то, что не было рассмотрено в классе.
  2. Обычно я стараюсь решать задачи в классе, которые отличаются от моих заметок. Тем не менее, с дифференциальным уравнением многие проблемы трудно решить под влиянием момента, и поэтому в этом классе моя классная работа будет следовать этим примечаниям довольно близко, насколько проработаны проблемы. С учетом сказанного, я буду время от времени решать проблемы наобум, когда смогу привести больше примеров, чем те, что есть в моих заметках. Кроме того, у меня часто нет времени в классе, чтобы решить все задачи в примечаниях, поэтому вы обнаружите, что некоторые разделы содержат задачи, которые не были решены в классе из-за ограничений по времени.
  3. Иногда вопросы в классе ведут по путям, которые здесь не рассматриваются. Я стараюсь предвидеть как можно больше вопросов, когда пишу их, но реальность такова, что я не могу предвидеть все вопросы. Иногда в классе задают очень хороший вопрос, который приводит к идеям, которые я здесь не включил. Вы всегда должны поговорить с кем-то, кто был в классе в тот день, когда вы пропустили, и сравнить эти записи с их заметками и посмотреть, в чем различия.
  4. Это несколько связано с предыдущими тремя пунктами, но достаточно важно, чтобы заслужить отдельный пункт. ЭТИ ЗАПИСКИ НЕ ЗАМЕНЯЮТ ПОСЕЩЕНИЕ ЗАНЯТИЙ!! Использование этих заметок вместо урока может привести к неприятностям. Как уже отмечалось, не все в этих заметках освещается в классе, и часто материал или идеи, не указанные в этих заметках, рассматриваются в классе.

Вот список (и краткое описание) материалов, содержащихся в этом наборе заметок.

Основные понятия. В этой главе мы вводим многие из основных понятий и определений, которые встречаются в типичном курсе дифференциальных уравнений. Мы также рассмотрим поля направлений и то, как их можно использовать для определения поведения решений дифференциальных уравнений.

Определения. В этом разделе вводятся некоторые из общих определений и понятий курса дифференциальных уравнений, включая порядок, линейные и нелинейные уравнения, начальные условия, задачу с начальными значениями и интервал достоверности.
Поля направлений – В этом разделе мы обсудим поля направлений и способы их наброска. Мы также исследуем, как можно использовать поля направлений для получения некоторой информации о решении дифференциального уравнения, не имея фактического решения.
. Заключительные мысли. В этом разделе мы даем пару заключительных мыслей о том, что мы будем рассматривать на протяжении всего курса.

Дифференциальные уравнения первого порядка. В этой главе мы рассмотрим несколько стандартных методов решения дифференциальных уравнений первого порядка, включая линейные, разделимые, точные дифференциальные уравнения и уравнения Бернулли. Мы также рассмотрим интервалы достоверности, равновесные решения и метод Эйлера. Кроме того, мы моделируем некоторые физические ситуации с помощью дифференциальных уравнений первого порядка.

Линейные уравнения. В этом разделе мы решаем линейные дифференциальные уравнения первого порядка, то есть дифференциальные уравнения в форме \(y’ + p(t) y = g(t)\). Мы даем подробный обзор процесса, используемого для решения этого типа дифференциального уравнения, а также вывод формулы, необходимой для интегрирующего коэффициента, используемого в процессе решения.
Разделимые уравнения. В этом разделе мы решаем разделимые дифференциальные уравнения первого порядка, то есть дифференциальные уравнения в форме \(N(y) y’ = M(x)\). Мы дадим вывод процесса решения этого типа дифференциального уравнения. Мы также начнем искать интервал достоверности решения дифференциального уравнения. 9{н}\). В этом разделе также будет представлена ​​идея использования подстановки для решения дифференциальных уравнений.
Замены. В этом разделе мы продолжим с того места, где остановился последний раздел, и рассмотрим пару других замен, которые можно использовать для решения некоторых дифференциальных уравнений. В частности, мы обсудим использование решений для решения дифференциальных уравнений вида \(y’ = F(\frac{y}{x})\) и \(y’ = G(ax + by)\).
Интервалы достоверности. В этом разделе мы подробно рассмотрим интервалы достоверности, а также ответим на вопрос о существовании и уникальности дифференциальных уравнений первого порядка.
Моделирование с помощью дифференциальных уравнений первого порядка. В этом разделе мы будем использовать дифференциальные уравнения первого порядка для моделирования физических ситуаций. В частности, мы рассмотрим задачи смешивания (моделирование количества вещества, растворенного в жидкости, и жидкости, которая входит и выходит), проблемы населения (моделирование населения в различных ситуациях, в которых население может войти или выйти) и падающие предметы. (моделирование скорости падающего объекта под действием силы тяжести и сопротивления воздуха).
Равновесные решения. В этом разделе мы определим равновесные решения (или точки равновесия) для автономных дифференциальных уравнений \(y’ = f(y)\). Мы обсуждаем классификацию равновесных решений как асимптотически устойчивых, неустойчивых и полуустойчивых равновесных решений.
Метод Эйлера. В этом разделе мы кратко рассмотрим довольно простой метод аппроксимации решений дифференциальных уравнений. Мы выводим формулы, используемые методом Эйлера, и даем краткое обсуждение ошибок в аппроксимациях решений.

Дифференциальные уравнения второго порядка. В этой главе мы начнем рассмотрение дифференциальных уравнений второго порядка. Мы сосредоточимся главным образом на дифференциальных уравнениях второго порядка с постоянными коэффициентами. Мы будем получать решения для однородных дифференциальных уравнений и будем использовать методы неопределенных коэффициентов и варьирования параметров для решения неоднородных дифференциальных уравнений. Кроме того, мы обсудим снижение порядка, основы множеств решений, вронскиан и механические колебания.

Основные понятия. В этом разделе дайте подробное обсуждение процесса, используемого для решения однородных линейных дифференциальных уравнений второго порядка \(ay» + by’ + cy = 0\). { 2} + br + c = 0\) — вещественные различные корни. 9{2} + br + c = 0\), повторяются, т. е. двойных, корней. Мы будем использовать понижение порядка, чтобы получить второе решение, необходимое для получения общего решения в этом случае.
Понижение порядка. В этом разделе мы более подробно обсудим понижение порядка, процесс, используемый для получения решения случая повторяющихся корней для однородных линейных дифференциальных уравнений второго порядка. Это будет один из немногих случаев в этой главе, когда будет рассмотрено дифференциальное уравнение с непостоянными коэффициентами.
Фундаментальные наборы решений. В этом разделе мы рассмотрим некоторые теории, лежащие в основе решения дифференциальных уравнений второго порядка. Мы определяем фундаментальные наборы решений и обсуждаем, как их можно использовать для получения общего решения однородного дифференциального уравнения второго порядка. Мы также определим вронскиан и покажем, как его можно использовать, чтобы определить, является ли пара решений фундаментальным набором решений.
Еще о вронскиане. В этом разделе мы рассмотрим, как можно использовать вронскиан, введенный в предыдущем разделе, для определения того, являются ли две функции линейно независимыми или линейно зависимыми. Мы также дадим и альтернативный метод нахождения вронскиана.
Неоднородные дифференциальные уравнения. В этом разделе мы обсудим основы решения неоднородных дифференциальных уравнений. Определим дополнительное и частное решение и приведем форму общего решения неоднородного дифференциального уравнения.
Неопределенные коэффициенты. В этом разделе мы вводим метод неопределенных коэффициентов для нахождения частных решений неоднородного дифференциального уравнения. Мы работаем с большим количеством примеров, иллюстрирующих множество рекомендаций по первоначальному предположению о форме конкретного решения, необходимого для метода.
Изменение параметров – В этом разделе мы вводим метод изменения параметров для нахождения частных решений неоднородного дифференциального уравнения. Мы даем подробное рассмотрение метода, а также выводим формулу, по которой можно найти частные решения.
Механические вибрации. В этом разделе мы рассмотрим механические вибрации. В частности, мы будем моделировать объект, соединенный с пружиной и движущийся вверх и вниз. Мы также допускаем введение в систему демпфера и действия общих внешних сил на объект. Также обратите внимание, что, хотя в этом разделе мы приводим примеры механических вибраций, простое изменение обозначений (и соответствующее изменение в том, что представляют величины) может перенести это практически в любую другую область техники.

Преобразования Лапласа. В этой главе мы познакомимся с преобразованиями Лапласа и тем, как они используются для решения задач с начальными значениями. С введением преобразований Лапласа мы теперь сможем решать некоторые задачи с начальными значениями, которые мы не смогли бы решить иначе. Мы будем решать дифференциальные уравнения, в которых используются дельта-функции Хевисайда и Дирака. Мы также дадим краткий обзор использования преобразований Лапласа для решения дифференциальных уравнений с непостоянными коэффициентами. Кроме того, мы определим интеграл свертки и покажем, как его можно использовать для обратных преобразований.

Определение. В этом разделе мы даем определение преобразования Лапласа. Мы также вычислим пару преобразований Лапласа, используя определение.
Преобразования Лапласа. В этом разделе мы познакомим вас с тем, как мы обычно вычисляем преобразования Лапласа, что позволяет избежать необходимости использовать определение. Мы обсудим таблицу преобразований Лапласа, использованную в этом материале, и проработаем множество примеров, иллюстрирующих использование таблицы преобразований Лапласа.
Обратные преобразования Лапласа. В этом разделе мы задаем вопрос, противоположный предыдущему. Другими словами, учитывая преобразование Лапласа, какая функция у нас изначально была? Мы снова работаем с множеством примеров, иллюстрирующих, как использовать таблицу преобразований Лапласа для этого, а также некоторые манипуляции с данным преобразованием Лапласа, необходимые для использования таблицы.
Ступенчатые функции. В этом разделе мы вводим ступенчатую функцию или функцию Хевисайда. Мы проиллюстрируем, как записать кусочную функцию в терминах функций Хевисайда. Мы также работаем с множеством примеров, показывающих, как использовать преобразования Лапласа и обратные преобразования Лапласа, которые включают функции Хевисайда. Мы также выводим формулы преобразования Лапласа функций, включающих функции Хевисайда.
Решение IVP с помощью преобразований Лапласа. В этом разделе мы рассмотрим, как использовать преобразования Лапласа для решения IVP. Примеры в этом разделе ограничены дифференциальными уравнениями, которые можно решить без использования преобразования Лапласа. Преимущество начала с этого типа дифференциального уравнения состоит в том, что работа, как правило, не такая сложная, и мы всегда можем проверить свои ответы, если захотим.
IVP с непостоянными коэффициентами. В этом разделе мы дадим краткий обзор использования преобразований Лапласа для решения IVP с непостоянными коэффициентами. Мы не работаем с большим количеством примеров в этом разделе. Мы работаем только с парой, чтобы проиллюстрировать, как работает процесс с преобразованиями Лапласа.
IVP со ступенчатыми функциями — это раздел, в котором причина использования преобразований Лапласа действительно становится очевидной. Мы будем использовать преобразования Лапласа для решения IVP, которые содержат функции Хевисайда (или ступенчатые). Без преобразований Лапласа их решение потребовало бы немало работы. Хотя мы работаем с одним из этих примеров без преобразований Лапласа, мы делаем это только для того, чтобы показать, что было бы, если бы мы попытались решить один из примеров без использования преобразований Лапласа.
Дельта-функция Дирака. В этом разделе мы вводим дельта-функцию Дирака и получаем преобразование Лапласа дельта-функции Дирака. Мы работаем с несколькими примерами решения дифференциальных уравнений, включающих дельта-функции Дирака, и, в отличие от задач с функциями Хевисайда, наш единственный реальный вариант для такого рода дифференциальных уравнений — использовать преобразования Лапласа. Мы также даем хорошее соотношение между дельта-функциями Хевисайда и Дирака.
Интеграл свертки. В этом разделе мы даем краткое введение в интеграл свертки и то, как его можно использовать для выполнения обратных преобразований Лапласа. Мы также проиллюстрируем его использование при решении дифференциального уравнения, в котором вынуждающая функция ( т. е. термин без y в нем) неизвестен.
Таблица преобразований Лапласа. Этот раздел представляет собой таблицу преобразований Лапласа, которую мы будем использовать в материале. Мы даем как можно более широкий спектр преобразований Лапласа, включая те, которые не часто приводятся в таблицах преобразований Лапласа.

Системы дифференциальных уравнений. В этой главе мы рассмотрим решение систем дифференциальных уравнений. Мы ограничимся системами двух линейных дифференциальных уравнений для целей обсуждения, но многие методы будут распространены на более крупные системы линейных дифференциальных уравнений. Мы также исследуем наброски фазовых плоскостей/портретов для систем двух дифференциальных уравнений. Кроме того, мы даем краткие обсуждения использования преобразований Лапласа для решения систем и некоторых моделей моделирования, которые приводят к системам дифференциальных уравнений.

Обзор: Системы уравнений. В этом разделе мы дадим обзор традиционной отправной точки для класса линейной алгебры. Мы будем использовать методы линейной алгебры для решения системы уравнений, а также приведем несколько полезных фактов о количестве решений, которые может иметь система уравнений. Обзор
: Матрицы и векторы. В этом разделе мы дадим краткий обзор матриц и векторов. Мы рассмотрим арифметику с использованием матриц и векторов, нахождение обратной матрицы, вычисление определителя матрицы, линейно зависимые/независимые векторы и преобразование систем уравнений в матричную форму.
Обзор: Собственные значения и собственные векторы. В этом разделе мы познакомимся с понятием собственных значений и собственных векторов матрицы. { \text{th}}\) в систему дифференциальных уравнений.
Решения для систем. В этом разделе мы кратко рассмотрим, как мы решаем системы дифференциальных уравнений в матричной форме. Мы также определяем вронскиан для систем дифференциальных уравнений и показываем, как его можно использовать для определения наличия общего решения системы дифференциальных уравнений.
Phase Plane – В этом разделе мы дадим краткое введение в фазовую плоскость и фазовые портреты. Мы определяем равновесное решение/точку для однородной системы дифференциальных уравнений и то, как можно использовать фазовые портреты для определения устойчивости равновесного решения. Мы также показываем формальный метод построения фазовых портретов.
Вещественные собственные значения. В этом разделе мы будем решать системы двух линейных дифференциальных уравнений, в которых собственные значения являются различными действительными числами. Мы также покажем, как рисовать фазовые портреты, связанные с реальными различными собственными значениями (седловыми точками и узлами).
Комплексные собственные значения. В этом разделе мы будем решать системы двух линейных дифференциальных уравнений, в которых собственные значения являются комплексными числами. Это будет включать в себя иллюстрацию того, как получить решение, которое не включает комплексные числа, которые мы обычно ищем в этих случаях. Мы также покажем, как рисовать фазовые портреты, связанные со сложными собственными значениями (центрами и спиралями).
Повторяющиеся собственные значения. В этом разделе мы будем решать системы двух линейных дифференциальных уравнений, в которых собственные значения представляют собой действительные повторяющиеся (в данном случае двойные) числа. Это будет включать получение второго линейно независимого решения, которое нам понадобится для формирования общего решения системы. Мы также покажем, как рисовать фазовые портреты, связанные с реальными повторяющимися собственными значениями (неправильными узлами).
Неоднородные системы. В этом разделе мы рассмотрим краткие примеры, иллюстрирующие использование неопределенных коэффициентов и вариаций параметров для решения неоднородных систем дифференциальных уравнений. Метод неопределенных коэффициентов будет работать почти так же, как и для дифференциальных уравнений n-го порядка, в то время как изменение параметров потребует дополнительной работы по выводу, чтобы получить формулу/процесс, который мы можем использовать в системах.
Преобразования Лапласа. В этом разделе мы рассмотрим быстрый пример, иллюстрирующий, как можно использовать преобразования Лапласа для решения системы двух линейных дифференциальных уравнений.
Моделирование. В этом разделе мы кратко рассмотрим некоторые расширения некоторых моделей моделирования, которые мы использовали в предыдущих главах и которые приводят к системам дифференциальных уравнений. В частности, мы рассмотрим задачи смешивания, в которых у нас есть два соединенных резервуара с водой, задачу хищник-жертва, в которой учитываются популяции обоих, и задачу о механических вибрациях с двумя массами, соединенными с пружиной, каждая из которых соединена с стена с пружиной.

Series Решения дифференциальных уравнений. В этой главе мы кратко рассмотрим, как представить решение дифференциального уравнения с помощью степенного ряда. Мы также рассмотрим, как решить дифференциальное уравнение Эйлера. Кроме того, мы сделаем краткий обзор степенных рядов и рядов Тейлора, чтобы помочь в работе с этой главой.

Обзор: Power Series – В этом разделе мы даем краткий обзор некоторых основ Power Series. Включены обсуждения использования теста отношения, чтобы определить, будет ли сходиться степенной ряд, сложение/вычитание степенного ряда, дифференциация степенного ряда и индексные сдвиги для степенного ряда. 9{x}\) и \(\cos(x)\) о \(x = 0\), а также показано, как записать ряд Тейлора для многочлена. Решения серии
. В этом разделе мы определяем обычные и особые точки для дифференциального уравнения. Мы также показываем, кто строит решение ряда дифференциального уравнения относительно обыкновенной точки. Метод, проиллюстрированный в этом разделе, полезен для решения или, по крайней мере, получения аппроксимации решения дифференциальных уравнений с непостоянными коэффициентами. 9{2}y» + b x y’ + c y = 0\). Обратите внимание, что, хотя это не связано с решением ряда, оно включено в главу о решении ряда, поскольку иллюстрирует, как получить решение по крайней мере одного типа дифференциального уравнения в особой точке.

Дифференциальные уравнения более высокого порядка. В этой главе мы рассмотрим распространение многих идей предыдущих глав на дифференциальные уравнения с порядком выше 2-го порядка. В некоторых случаях это будет просто означать работу на примере, чтобы проиллюстрировать, что процесс на самом деле не меняется, но в большинстве случаев есть некоторые вопросы для обсуждения. 9{\text{th}}\) Линейные уравнения порядка. В этом разделе мы начнем главу с краткого обзора некоторых основных идей, лежащих в основе решения линейных дифференциальных уравнений высшего порядка. Включены будут обновленные определения/факты для принципа суперпозиции, линейно независимых функций и вронскиана.
Линейные однородные дифференциальные уравнения. В этом разделе мы расширим идеи решения линейных однородных дифференциальных уравнений 2 -го и -го порядков до более высокого порядка. Как мы увидим, большая часть процесса идентична нескольким естественным расширениям повторяющихся действительных корней, которые встречаются более двух раз. Нам также нужно будет обсудить, как поступать с повторяющимися сложными корнями, которые теперь возможны. Кроме того, мы увидим, что основная трудность в случаях высших порядков состоит в простом нахождении всех корней характеристического многочлена.
Неопределенные коэффициенты. В этом разделе мы рассмотрим быстрый пример, чтобы проиллюстрировать, что использование неопределенных коэффициентов в дифференциальных уравнениях более высокого порядка ничем не отличается от того, когда мы использовали его в дифференциальных уравнениях 2 -го и -го порядков только с одним небольшим естественным расширением.
Вариация параметров. В этом разделе мы дадим подробное обсуждение процесса использования вариации параметров для дифференциальных уравнений высшего порядка. Мы также разработаем формулу, которую можно использовать в этих случаях. Мы также увидим, что работа, связанная с использованием вариации параметров в дифференциальных уравнениях более высокого порядка, может иногда быть весьма сложной.
Преобразования Лапласа. В этом разделе мы рассмотрим быстрый пример использования преобразований Лапласа для решения дифференциального уравнения с дифференциальным уравнением порядка 3 rd просто для того, чтобы сказать, что мы рассмотрели уравнение с порядком выше 2 rd . Как мы увидим, помимо формулы для преобразования Лапласа \(y»’\), которую мы можем получить из общей формулы, нет никакой реальной разницы в том, как преобразования Лапласа используются для дифференциальных уравнений более высокого порядка. .
Системы дифференциальных уравнений. В этом разделе мы кратко рассмотрим расширение идей, которые мы обсуждали для решения систем дифференциальных уравнений \(2 \times 2\), на системы размера \(3 \times 3\). Как мы увидим, в основном это просто естественные расширения того, что мы уже знаем, что делать. Мы также сделаем несколько быстрых замечаний о системах \(4 \times 4\). 9Решения серии 0003. В этом разделе мы рассмотрим быстрый пример, показывающий, что процесс поиска решений серии для дифференциальных уравнений более высокого порядка почти такой же, как и для дифференциальных уравнений 2 -го и -го порядков.

Краевые задачи и ряды Фурье. В этой главе мы представим две темы, которые являются неотъемлемой частью основных методов решения дифференциальных уравнений в частных производных. Первая тема, краевые задачи, встречается практически в каждом уравнении с частными производными. Вторая тема, ряды Фурье, — это то, что заставляет работать один из основных методов решения.

Краевые задачи. В этом разделе мы определим граничные условия (в отличие от начальных условий, с которыми мы уже должны быть знакомы на этом этапе) и краевую задачу. Мы также рассмотрим несколько примеров, иллюстрирующих некоторые интересные различия в использовании граничных значений вместо начальных условий при решении дифференциальных уравнений. \infty {{B_n}\sin\left( {\ frac{{n\pi x}}{L}} \right)} \). Мы также определим нечетное расширение для функции и поработаем с несколькими примерами, находя ряд синусов Фурье для функции. 9\infty {{B_n}\sin\left( {\frac{{n\pi x}}{L}} \right)} \). Мы также рассмотрим несколько примеров нахождения ряда Фурье для функции.
Сходимость рядов Фурье. В этом разделе мы определим кусочно-гладкие функции и периодическое расширение функции. Кроме того, мы приведем множество фактов о том, к чему будет сходиться ряд Фурье и когда мы можем ожидать сходимости производной или интеграла ряда Фурье к производной или интегралу функции, которую он представляет.

Уравнения в частных производных. В этой главе мы познакомим вас с разделением переменных, одним из основных методов решения уравнений в частных производных. Включены частичные выводы для уравнения теплопроводности и волнового уравнения. Кроме того, мы приводим решения примеров для уравнения теплопроводности, волнового уравнения и уравнения Лапласа.

Уравнение теплопроводности. В этом разделе мы сделаем частичный вывод уравнения теплопроводности, которое можно решить для получения температуры в одномерном стержне длиной L. Кроме того, мы даем несколько возможных граничных условий, которые можно использовать в этой ситуации. . В этом разделе мы также определяем лапласиан и даем версию уравнения теплопроводности для двух- или трехмерных ситуаций.
Волновое уравнение. В этом разделе мы делаем частичный вывод волнового уравнения, которое можно использовать для нахождения одномерного смещения вибрирующей струны. Кроме того, мы также даем двухмерную и трехмерную версии волнового уравнения.
Терминология. В этом разделе мы кратко рассмотрим некоторые термины, которые будем использовать в оставшейся части этой главы. В частности, мы определим линейный оператор, линейное уравнение в частных производных и однородное уравнение в частных производных. Мы также даем краткое напоминание о принципе суперпозиции.
Разделение переменных. В этом разделе показано, как метод разделения переменных можно применить к уравнению в частных производных, чтобы свести уравнение в частных производных к двум обыкновенным дифференциальным уравнениям. Мы применяем метод к нескольким уравнениям в частных производных. Мы, однако, не идем дальше в процессе решения дифференциальных уравнений в частных производных. Это будет сделано в последующих разделах. Смысл этого раздела только в том, чтобы проиллюстрировать, как работает метод.
Решение уравнения теплопроводности. В этом разделе мы рассмотрим процесс полного разделения переменных, включая решение двух обыкновенных дифференциальных уравнений, порождаемых процессом. Мы сделаем это, решив уравнение теплопроводности с тремя различными наборами граничных условий. Включен пример решения уравнения теплопроводности на стержне длиной \(L\), но вместо этого на тонком круглом кольце.
Уравнение теплопроводности с ненулевыми температурными границами. В этом разделе мы кратко рассмотрим решение уравнения теплопроводности, в котором граничные условия фиксированы, ненулевая температура. Обратите внимание, что это отличается от предыдущего раздела, когда мы обычно требовали, чтобы граничные условия были фиксированными и нулевыми.
Уравнение Лапласа. В этом разделе мы обсудим решение уравнения Лапласа. Как мы увидим, это именно то уравнение, которое нам нужно было бы решить, если бы мы хотели найти равновесное решение (, т.е. , не зависящее от времени) для двумерного уравнения теплопроводности без источников. Мы также переведем уравнение Лапласа в полярные координаты и решим его на круге радиуса \(a\).
Вибрирующая струна. В этом разделе мы решим одномерное волновое уравнение, чтобы получить смещение вибрирующей струны.
Краткое изложение метода разделения переменных. В этом последнем разделе мы даем краткий обзор метода разделения переменных для решения уравнений в частных производных.


Дифференциальные уравнения для чайников Памятка

Чтобы уверенно решать дифференциальные уравнения, необходимо понимать, как уравнения классифицируются по порядку, как различать линейные, разделимые и точные уравнения, а также как определять однородные и неоднородные дифференциальные уравнения.

Изучить метод неопределенных коэффициентов для решения неоднородных дифференциальных уравнений.

Классификация дифференциальных уравнений по порядку

Наиболее распространенная классификация дифференциальных уравнений основана на порядке. Порядок дифференциального уравнения — это просто порядок его старшей производной. У вас могут быть дифференциальные уравнения первого, второго и более высокого порядка.

Первые дифференциальные уравнения порядка включают производные первого порядка, как в этом примере:

Второй дифференциальные уравнения порядка включают производные второго порядка, например, в этих примерах:

Высшее Дифференциальные уравнения порядка – это те, которые содержат производные выше второго порядка (большой сюрприз для такого умного названия!). Дифференциальные уравнения всех порядков могут использовать обозначение y ’, например:

Отличие линейных, разделимых и точных дифференциальных уравнений

Вы можете различать линейные, разделимые и точные дифференциальные уравнения, если знаете, что искать. Имейте в виду, что вам может потребоваться перетасовать уравнение, чтобы идентифицировать его.

Линейные дифференциальные уравнения включают только производные y и члены y в первой степени, не возведенные в более высокую степень. ( Примечание: Это степень, в которой производная возведена в , а не порядка производной.) Например, это линейное дифференциальное уравнение, поскольку оно содержит только производные, возведенные в первую степень:

S разделимые дифференциальные уравнения можно записать так, что все члены в x и все члены в y появляются на противоположных сторонах уравнения. Вот пример:

, которое с небольшой перетасовкой можно записать так:

Точные дифференциальные уравнения позволяют найти функцию, частные производные которой соответствуют членам данного дифференциального уравнения.

Определение однородных и неоднородных дифференциальных уравнений

Чтобы идентифицировать неоднородное дифференциальное уравнение, сначала нужно знать, как выглядит однородное дифференциальное уравнение. Вам также часто нужно решить одну, прежде чем вы сможете решить другую.

Однородные дифференциальные уравнения включают только производные от y и члены, включающие y , и они установлены равными 0, как в этом уравнении:

Неоднородные дифференциальные уравнения аналогичны однородным дифференциальным уравнениям, за исключением того, что они могут иметь члены, включающие только x (и константы) справа, как в этом уравнении:

You also can write nonhomogeneous differential equations in this format: y ” + p ( x ) y ‘ + q ( x ) y = g ( x ) . Общее решение этого неоднородного дифференциального уравнения равно

.

В этом растворе c 1 y 1 ( x ) + c 2 y 2 ( x ) является общим решением соответствующего однородного дифференциального уравнения:

А y p ( x ) является частным решением неоднородного уравнения.

Использование метода неопределенных коэффициентов

Если вам необходимо найти частные решения неоднородных дифференциальных уравнений, то вы можете начать с метода неопределенных коэффициентов. Предположим, вы столкнулись со следующим неоднородным дифференциальным уравнением:

Метод неопределенных коэффициентов отмечает, что когда вы найдете возможное решение y и подставите его в левую часть уравнения, вы получите g ( x ). Поскольку g ( x ) является только функцией x , вы часто можете угадать форму y p ( x ) с точностью до произвольных коэффициентов, а затем найти эти коэффициенты. заглушкой y p ( x ) в дифференциальное уравнение.

Этот метод работает, потому что вы имеете дело только с г ( x ), а форма г ( x ) часто может сказать вам, как выглядит конкретное решение.

Об этой статье

Эта статья взята из книги:

  • Дифференциальные уравнения для чайников,

Об авторе книги:

Стивен Хольцнер — отмеченный наградами автор книг по естественным наукам, математике и технике. Он изучал дифференциальные уравнения в Массачусетском технологическом институте и в Корнельском университете, где получил степень доктора философии. Он работал преподавателем в Массачусетском технологическом институте и Корнельском университете и написал такие бестселлеры, как Физика для чайников и Рабочая тетрадь по физике для чайников.

Эту статью можно найти в категории:

  • Исчисление,

Обработка решений · DifferentialEquations.

jl

Тип решения имеет множество встроенных функций, помогающих в анализе. Например, он имеет интерфейс массива для доступа к значениям. Внутри тип решения имеет два важных поля:

  1. u , которые содержат вектор значений на каждом временном шаге 9.0082
  2. t , который содержит время каждого временного шага.

Различные типы решений могут при необходимости добавлять дополнительную информацию, такую ​​как производная на каждом временном шаге du или пространственная дискретизация x , y и т. д.

Вместо работы с Vector{uType} напрямую, мы можем использовать предоставленный интерфейс массива.

 sol[j] 

для доступа к значению на временном шаге j (если временной ряд был сохранен) и

 sol.t[j] 

для доступа к значению t на временном шаге j . Для многомерных систем это будет адресоваться сначала по компонентам, а затем по времени, и, таким образом,

 sol[i,j] 

будет i -м компонентом на временном шаге j . Следовательно, sol[j][i] == sol[i, j] . Это сделано потому, что Джулия является главным столбцом, поэтому ведущее измерение должно быть непрерывным в памяти. Если бы независимые переменные имели форму (например, были матрицей), то i — линейный индекс. Мы также можем получить доступ к решениям с формой:

 sol[i,k,j] 

дает компонент [i,k] системы на временном шаге j . Поддерживается оператор двоеточия, означающий, что

 sol[i,:] 

дает временной ряд для i -го компонента.

Интерфейс AbstractArray можно использовать напрямую. Например, для векторной системы переменных sol[i,j] — это матрица, в которой строки являются переменными, а столбцы — моментами времени. Операции типа sol' изменит тип решения. Функциональность, написанная для AbstractArray , может напрямую использовать это. Например, функция Base cov вычисляет корреляции между столбцами, и таким образом:

 cov(sol) 

вычисляет корреляцию состояния системы во времени, тогда как

 cov(sol,2) 

вычисляет корреляцию между переменные. Точно так же mean(sol,2) является средним значением переменной во времени, а var(sol,2) это дисперсия. Другие статистические функции и пакеты, которые работают с типами AbstractArray , будут работать и с типом решения.

В любое время можно создать настоящий массив с помощью Array(sol) .

Если решатель допускает плотный вывод и для решения было установлено плотное=истинное (по умолчанию), то мы можем получить приблизительное значение за время t с помощью команды

 sol(t) 

Обратите внимание, что функция интерполяции допускает t в качестве вектора и использует его для ускорения вычислений интерполяции. Полный API для интерполяции:

 sol(t,deriv=Val{0};idxs=nothing,continuity=:left) 

Необязательный аргумент deriv позволяет выбрать производную числа n для решения интерполяции. для, по умолчанию n=0 . Обратите внимание, что большинство производных еще не реализовано (хотя это не сложно, это просто нужно сделать вручную для каждого алгоритма. Если вам нужен какой-то конкретный, откройте вопрос). непрерывность описывает необходимость соблюдения непрерывности слева или справа при сохранении разрыва. Значение по умолчанию — :left , т. е. захват значения до изменения обратного вызова, но его можно изменить на :right . idxs позволяет вам выбрать индексы, для которых должна выполняться интерполяция. Например,

 sol(t,idxs=1:2:5) 

вернет вектор длины 3, который представляет собой интерполированные значения t для компонентов 1 , 3 и 5 . idxs=nothing , значение по умолчанию, означает, что будут возвращены все компоненты. Кроме того, мы можем сделать

 sol(t,idxs=1) 

, и он вернет число для интерполяции одного значения. Обратите внимание, что эта интерполяция вычисляет только запрошенные значения, и поэтому в больших системах гораздо быстрее использовать ее, чем вычислять полную интерполяцию и использовать только несколько значений.

Кроме того, есть замещающая форма:

 sol(out,t,deriv=Val{0};idxs=nothing,continuity=:left) 

, который запишет вывод в из . Это позволяет использовать предварительно выделенные векторы для вывода, чтобы еще больше повысить скорость.

Интерфейс решателя также предоставляет инструменты для использования вкраплений в решении. Используя функцию tuples(sol) , мы можем получить кортеж для вывода на каждом временном шаге. Это позволяет сделать следующее:

 [t+2u for (u,t) in tuples(sol)] 

Можно использовать дополнительные компоненты объекта решения, а также использовать zip . Например, предположим, что тип решения содержит du , производную на каждом временном шаге. Можно понять значения, используя:

 [t+3u-du for (t,u,du) in zip(sol. t,sol.u,sol.du)] 

Обратите внимание, что объект решения действует как вектор во времени, поэтому его длина равна количеству сохраненных моментов времени.

Интерфейс решения также содержит несколько специальных полей. Проблемный объект prob и алгоритм, использованный для решения задачи alg , включены в решение. Кроме того, поле плотное является логическим значением, указывающим, доступны ли функции интерполяции. Кроме того, поле destats содержит внутреннюю статистику для процесса решения, такую ​​как количество линейных решений и ошибок сходимости. Наконец, есть изменяемое состояние tslocation , которое управляет поведением рецепта графика. По умолчанию tslocation=0 . Его значения имеют разные значения для уравнений в частных и обыкновенных дифференциальных уравнениях:

  • tslocation=0 для непространственных задач (ОДУ) означает, что рецепт построения графика будет отображать полное решение. tslocation=i означает, что будет построена только точка времени i .
  • tslocation=0 для пространственных задач (PDE) означает, что рецепт графика будет отображать конечную точку времени. tslocation=i означает, что рецепт печати будет отображать i -й момент времени.

Это означает, что для ODE графики по умолчанию будут представлять собой полный график, а PDE по умолчанию будут отображать поверхность в последний момент времени. Интерфейс итератора просто повторяет значение tslocation , а функция animate повторяет решение, вызывающее решение на каждом шаге.

DiffEqBase.DEStats — Тип
 mutable struct DEStats 

Статистика из программы решения дифференциальных уравнений о процессе решения.

Поля

  • nf: Количество оценок функции. Если дифференциальное уравнение представляет собой расщепленную функцию, такую ​​как SplitFunction для неявно-явной (IMEX) интеграции, то nf — это число вычислений функции для первой функции (неявной функции)
  • nf2: если дифференциальное уравнение представляет собой функцию разделения, такую ​​как SplitFunction для неявно-явного (IMEX) интегрирования, тогда nf2 — это количество вычислений функции для второй функции, т. е. функция, обработанная явно. В противном случае он равен нулю.
  • nw: Количество матриц W=I-gamma*J (или W=I/gamma-J), построенных в процессе решения.
  • nsolve: Количество линейных решений W , необходимых для интегрирования.
  • njacs: количество якобианов, вычисленное при интегрировании.
  • nnonliniter: Общее количество итераций для нелинейных решателей.
  • nnonlinconvfail: Количество ошибок сходимости нелинейного решателя.
  • ncondition: количество вызовов функции условия для обратных вызовов.
  • naccept: Количество принятых шагов.
  • nreject: количество отклоненных шагов.
  • maxeig: максимальное собственное значение решения. Это вычисляется только в том случае, если метод является алгоритмом автоматического переключения.

Типы решений имеют поле retcode , которое возвращает символ, обозначающий состояние ошибки решения. Реткоды следующие:

  • : По умолчанию : Решатель не устанавливал реткоды.
  • :Успех : Интеграция завершена без ошибок или решатель устойчивого состояния с SteadyStateDiffEq обнаружил устойчивое состояние.
  • : Прекращено : Интеграция завершена с завершением!(интегратор) . Обратите внимание, что это может произойти при использовании TerminateSteadyState из библиотеки обратного вызова DiffEqCallbacks .
  • :MaxIters : Интеграция завершилась досрочно, так как достигнуто максимальное количество итераций.
  • :DtLessThanMin : Метод временного шага выбрал размер шага, который меньше допустимого минимального временного шага, и завершил работу досрочно.
  • :Нестабильно : Решатель обнаружил, что решение нестабильно, и завершил работу досрочно.
  • :InitialFailure : Решателю DAE не удалось найти согласованные начальные условия.
  • :ConvergenceFailure : Не удалось сойтись внутренним неявным решателям.

Добавить комментарий

Ваш адрес email не будет опубликован. Обязательные поля помечены *